NCLEX HURST 2/13/22

Pataasin ang iyong marka sa homework at exams ngayon gamit ang Quizwiz!

The pediatric nurse is preparing to administer ciprofloxacin otic to a 2-year-old child. What action will the nurse take for proper administration? 1. Have the child look at the ceiling during administration. 2. Pull the ear down and back for administration. 3. Tilt the head to the affected side following administration. 4. Keep the eyes shut for 10 seconds after administration.

.CORRECT. For children less then 3 years of age, the ear needs to be pulled down and back. Over the age of 3, the ear is pulled up and back. 1. INCORRECT. The medication is going in the ear. The child does not have to look at the ceiling. This will also make it difficult to administer. 3. INCORRECT. If the head is tilted to the affected side following administration, the medication may drain out. 4. INCORRECT. Otic is for the ears. Closing the eyes has nothing to do with it.

An OB/GYN unit has recently discovered an increase in staphylococcus infections among both clients and staff, even though all cleaning procedures have been verified and upgraded. The infection control nurse is attempting to locate the source of the infection. The nurse knows which situations would likely not contribute to this problem? Select all that apply 1. A client in isolation receiving meals on disposable trays and dishes. 2. A client awaiting induction with a bouquet of flowers in the room. 3. A housekeeper sharing cleaning supplies with nighttime personnel. 4. A nurse sharing samples of a new hand cream with staff and clients. 5. A client recovering from a hysterectomy with large number of visitors.

1, 2 & 3. Correct: Certain hospital policies are developed to help eliminate potential danger to the most vulnerable population of clients. Clients in isolation receive food on styrofoam trays with disposable containers and utensils. These items are placed into a biohazard container in the room for safe disposal. A client awaiting induction is permitted to have flowers unless otherwise instructed by the facility, doctor or unit. Cut flowers are usually safer than potted plants, which tend to carry bacteria. Sharing cleaning supplies among personnel does not present an infection hazard. 4. Incorrect: A potential source for infection is any moist environment which may create a medium for bacterial growth. Hand cream could be a possible source of infection, particularly if the nurse shares 'samples' from one community jar. 5. Incorrect: Any small environment, such as a hospital room, in which a large number of people congregate, could potentially create an infection hazard. Visitors may bring food or drink along with "treats" for the recovering client. Sometimes food is stored in drawers, or placed in closet and forgotten until bacteria has spread.

The nurse in the emergency department is caring for a client admitted in diabetic ketoacidosis (DKA). Which central venous pressure (CVP) reading would the nurse anticipate? 1. 1 mm of Hg 2. 3 mm of Hg 3. 6 mm of Hg 4. 12 mm of Hg

1. Correct: Normal CVP is 2-6 mmHg. This is a CVP reading that would indicate fluid volume deficit. A client in DKA will have polyuria. A Hurst strategy is "with polyuria, think shock first". Less volume, less pressure! 2. Incorrect: This is a normal CVP reading. Normal CVP is 2 to 6 mm of Hg. 3. Incorrect: This is a normal CVP reading. 4. Incorrect: This CVP reading indicates fluid volume overload. The client in DKA will not be experiencing fluid volume excess.

A nurse suspects that a client admitted to the emergency department is in diabetic ketoacidosis. What data would lead the nurse to this conclusion? Select all that apply 1. Dry mucous membranes 2. Fruity-smelling breath 3. Biot's respirations 4. Glycosuria 5. Client report of abdominal pain

1., 2., 4., & 5. Correct or True: The client with diabetic ketoacidosis will have signs of dehydration due to polyuria and includes dry mucous membranes. Fruity breath odor is from the acetone that occurs with breakdown of fats and formation of ketones, which are acids.. With DKA, the client would be spilling glucose into the urine. Vomiting and abdominal pain are frequently the presenting symptoms of DKA. 3. Incorrect or False: The client will have Kussmaul respirations. Biot's respiration is a respiratory pattern characterized by periods of rapid respirations, then apnea periods. These are not the type of respirations that occur with diabetic ketoacidosis (metabolic acidosis).

A nurse is teaching a group of expectant parents about epidural anesthesia. What information should the nurse include? Select all that apply 1. Contraindications for an epidural include a previous cesarean section. 2. Post procedure position should be side lying. 3. Headache is a post procedure side effect. 4. The major complication is hypotension. 5. Usually administered at 3-4 cm dilation.

2., 4., & 5. Correct: The client should not lie supine but should position self in a side-lying position. This will prevent compression on the vena cava. The major complication of epidural anesthesia is hypotension and supine position increases the risk. If this occurs, a bolus with 1000 mL of NS or LR to increase blood pressure by increasing vascular volume. Epidurals are usually placed during stage 1 at 3-4 cm dilation. 1. Incorrect: Previous C-sections do not eliminate the ability to have an epidural. Epidurals are commonly utilized for anesthesia during a cesarean birth. 3. Incorrect: A sterile guide needle and a small epidural catheter is placed between the spinal cord and the outer membrane. There is usually no headache since the needle does not enter the spinal column but rather the epidural space.

When making assignments for an LPN on the Labor and Delivery unit, the charge nurse is aware the most appropriate clients should meet what criteria? 1. Clients requiring close monitoring. 2. Post-vaginal delivery clients only. 3. Clients with a predictable outcome. 4. Non-routine clients in early labor.

3. Correct: A client with a predictable outcome is stable enough to be assigned to the licensed practical nurse (LPN). If complications should arise, the LPN would report this to the registered nurse. 1. Incorrect: Clients requiring close monitoring indicate an unstable or unpredictable status, which is not an appropriate assignment for the LPN. These clients should be assigned to a registered nurse. 2. Incorrect: The manner of delivery a client experienced does not dictate which staff personnel are able to provide care. More important factors would focus on whether the client had a non-eventful delivery or suffered any unexpected complications. 4. Incorrect: The term "non-routine" immediately indicates an unstable client with potential problems that need frequent assessment. An LPN should only be assigned to those stable clients with expected or predictable outcomes.

The nurse is teaching a group of clients in cardiac rehabilitation how blood flows through the heart. In what order should the nurse present this information? List the order in which blood flows through the heart, starting from deoxygenated blood in the body. Vena Cava Right Atrium Right Ventricle Lungs Left Atrium Left Ventricle Aorta

Deoxygenated blood comes from the body to the heart via the superior and inferior vena cava. From there blood enters the right atrium, then travels to the right ventricle. The right ventricle pumps the blood to the lungs via the pulmonary artery where the blood becomes oxygenated. From the lungs, oxygenated blood goes to the left atrium via the pulmonary vein, then to the left ventricle. The left ventricle pumps the blood out through the aorta to the body.

Place the steps in order that the nurse should take to administer a subcutaneous injection. Place the steps in order that the nurse should take to administer a subcutaneous injection. Perform hand hygiene Apply gloves and locate the injection site Cleanse site with antiseptic swab Remove the needle cap by pulling it straight off Hold syringe and pinch the skin with nondominant hand Inject the needle and administer the medication Dispose the syringe in sharps container

First perform hand hygiene. Then apply gloves and locate injection site using anatomical landmarks. Start at the center of the site and rotate outward in a circular direction to cleanse the site. Remove the needle cap by pulling the cap straight off. Next, hold the syringe and pinch the skin with nondominant hand. Inject the needle quickly then administer the medication slowly. Finally, dispose of the syringe in the sharps container.

A nurse utilizes the Braden Scale to assess and document the pressure sore risk of a client diagnosed with Guillain-Barré syndrome. Based on this documentation, what score should the nurse assign to this client? 0900 The client has very limited sensory perception and is occasionally moist. The client is wheelchair bound and has very limited mobility. The client is receiving adequate caloric intake by tube feeding and is moved easily by the nursing staff without any evidence of friction or sheering.

The correct answer is 15, placing this client at mild risk. Sensory perception very limited = 2 Moisture occasionally moist = 3 Chair fast = 2 Mobility very limited = 2 Adequate nutrition = 3 No friction/shear = 3 Total = 15 Braden Scale Assessment Score: Very High Risk: Total Score of 9 or less High Risk: Total Score of 10-12 Moderate Risk: Total Score of 13-14 Mild Risk: Total Score of 15-18 No Risk: Total Score of 19-23

Four clients arrive for their appointment at a diabetic clinic. In what order should the nurse see the clients? Client reporting a headache and has a fruity breath. Client eating a simple-carb snack due to weakness. Client scheduled for a dressing change to foot ulcer. Client to receive dietary education.

The first client needing the nurse's attention is the one reporting a headache and has a fruity odor to their breath. Remember, pick the killer answer first! This client is likely in metabolic acidosis due to diabetic ketoacidosis (DKA). What was the hint? Fruity breath. The second client that needs to be seen by the nurse is the client having weakness, a sign of hypoglycemia. This is a diabetic clinic. This client is eating a simple carb snack, but the nurse needs to check the client's blood glucose level to see if the snack has helped. The third client would be the one needing a dressing change. Nothing life threatening, but an assessment needs to be made regarding the ulcer. The last client would be the one needing dietary education. Nothing life threatening. This client can wait until the others are treated.

A nurse is caring for a group of clients and is considering the risk of infection for each. Place the client conditions in rank order from the highest to least potential for infection. The Correct Order Thermal burns covering 30% of body surface area (BSA) 2 days ago Total hip prosthetic device placement 3 days ago Laparoscopic exploration of right knee 2 days ago Indwelling foley catheter inserted the previous day

The client with the greatest risk of infection would be the client with thermal burns covering 30% of the BSA. Burns are considered contaminated wounds. Normally, skin provides a natural barrier against invasive microorganisms. However, with this major burn injury, the client is predisposed to infection as a result of the loss of skin integrity. Additional factors that will place this client at higher risk for infection include the development of eschar, which bacteria loves to live in, and the fact that thermal injuries alter the body's natural immunity. So, are the clients with the other conditions at risk for infection? Well, they could be, but the risk is not as great. Let's consider why the risk is less. The client with the total hip arthroplasty (replacement of the damaged hip with a prosthetic device implanted) would be the next highest in ranking for risk of infection. This client has a relatively large surgical incision and a prosthetic device that infection, when present, tends to migrate to the area. But, this type surgery is performed using sterile technique in sterile environments to minimize the risk of infection. In addition, any dressing changes should be performed using sterile technique. The next client at risk of infection would be the client with the laparoscopic exploration of the right knee. Again, there is surgical perforation of the skin. However, these are smaller puncture sites that are created under sterile conditions, and when cared for appropriately, do not carry a high risk for infection. Finally, the client who has the indwelling foley catheter is the least at risk for infection. The catheter is a portal of entry into the body, but if inserted using sterile technique and proper catheter care is provided, the risk of infection can be kept to a minimum. The longer the foley catheter remains in place, the risk of infection will increase.

The primary healthcare provider's prescription for a client instructs the nurse to give digoxin 0.125 mg intravenously as a one-time dose. The available medication is in a concentration of 0.5 mg/2 mL. How many milliliters should the nurse give? Round answer using one decimal point.

____ mL= 2 mL x 0.125 mg = 0.5 mL 0.5 mg

At a well-baby check, the parents of a 14 month old report how the child is doing and then excitedly share that they have purchased and are moving into a "fixer-up" home that was built in the mid-1960s. Based on the parent's report, what would be the priority concern for the nurse to address with the parents? 1. Fall risk due to increased mobility 2. Increased anxiety due to change in the environment 3. Speech consisting of only 4 words 4. Potential for lead poisoning

4. Correct: Since the home that they are moving into was built before 1978, there is a high likelihood that it has lead-based paint. One of the most common causes of lead poisoning is lead from paint, including dust that is contaminated with lead. Lead-based paint that is peeling, chipping, cracking, or damaged is a hazard that requires immediate attention. Young children, such as this 14 month old, are at high risk for lead poisoning as they may chew on painted surfaces or may come in contact with painted surfaces that often have areas of wear-and-tear such as window sills, door frames, railings, etc. In addition, remodeling or renovating activities as well as scraping old paint can create toxic lead dust. The reason that this is the priority concern is that even low levels of lead in the blood in children could result in learning and behavior problems, delayed growth, anemia, hearing problems, hyperactivity, and lower IQ. In more severe cases, lead ingestion can cause seizures, coma, and possibly death. 1. Incorrect: Yes, 14 month old children are likely to experience some tumbles and bumps. This is normal for this age child as depth perception and coordination are not fully developed and they are learning the skills of increased mobility. However, these are not generally of great concern under normal circumstances and certainly do not take priority over the concern for lead poisoning. 2. Incorrect: Toddlers tend to do better with routines that are predictable. Changes in the routine can cause the toddler to become anxious. Moving to a new home would be very disruptive to this 14 month old's routine. This anxiety could be manifested by increased clinginess, regression, decreased appetite, and other unusual behaviors such as increased aggressive behaviors, shyness, and anxious habits (twirling hair, clinging to and rubbing favorite blanket, etc.). Even though these behaviors may be concerning and would need to be addressed, they would not be a priority over the effects that lead poisoning could cause. 3. Incorrect: The speech of the 14 month old is still early in the developmental process and can vary from child to child. A 14 month old's vocabulary normally consists of about three to five words. So, this 14 month old with a vocabulary of 4 words is not abnormal and would not be of concern to the nurse.

The nurse is caring for a ventilator-dependent client assisted with positive expiratory end pressure (PEEP). The high-pressure alarm begins sounding. What actions should the nurse initiate? Select all that apply 1. Check to see if client is biting ET tube. 2. Examine tubing for presence of water. 3. Inspect for any loose connections. 4. Reduce the amount of PEEP used. 5. Assess client's need for suctioning.

1, 2 and 5. CORRECT: The high-pressure alarm on a ventilator indicates the machine is pushing against excessive resistance while trying to deliver oxygen to the client. There are multiple potential causes for a high pressure alarm. The client could be fighting against the ventilator and thus biting down on the endotracheal tube. Another possible issue may be the water that accumulates in the vent tubing from condensation. Or the client may have excessive mucus in the airways that requires the nurse to suction lungs frequently. Any of these problems could initiate the high pressure alarm. 3. INCORRECT: Loose or disconnected tubing results in a low pressure alarm, indicating the ventilator is unable to exert the expected amount of pressure needed to oxygenate client. 4. INCORRECT: The amount of PEEP provided to a ventilator-dependent client is determined by respiratory diagnosis or ABGs and is ordered by the primary healthcare provider. The high pressure alarm does not generally indicate a problem with the PEEP setting.

A client arrives at the emergency room with severe right foot pain and is admitted with a diagnosis of hyperuricemia (gout). The nurse is reviewing diet habits and life style with the client to develop a teaching care plan. The nurse has identified what habits that may contribute to an exacerbation of gout? Select all that apply 1. A daily glass of white wine. 2. Bacon and eggs on weekends. 3. Smoking two cigars every day. 4. One half liter of soda daily. 5. Baked cod twice a week. 6. A BMI of 31.5 kg/m2.

1, 4, 5 &6. Correct: Hyperuricemia is the presence of excessive uric acid crystals which lodge in or around joint spaces. Large amounts of purine in the body contribute to the development of the crystals. Alcohol, including wines and beer, increases these crystals as do sugary drinks such as soda pop. Red meats like liver, kidneys and even raw steak should be avoided along with specific fish such as cod, tuna, sardines and even anchovies. Another contributing factor is obesity. With an average BMI range of 24-25 kg/m2, depending on height, an adult client with a BMI of 30.5 kg/m2 is considered obese. 2. Incorrect: Though bacon is a meat product, pork is not as bad as red or organ meats. Eggs are beneficial and are not known to exacerbate gout. 3. Incorrect: Although smoking may affect many of the body functions, by itself smoking does not contribute to the development of uric acid crystals.

The nurse is caring for a client who has a history of sleep apnea. The client is scheduled for a colon resection the following morning and asks if the sleep apnea machine should be brought to the hospital. What is the nurse's best response? 1. Yes, bring the sleep apnea machine. 2. No, do not bring the sleep apnea machine. 3. It is your choice. 4. Call your primary healthcare provider.

1. Correct: A client with sleep apnea is at risk for cardiac and respiratory complications postop due to decreasing oxygenation. So yes, the client needs to use the CPAP machine. Remember this client will also be receiving narcotics for pain and have a decreased activity level as well. All of these things can decrease oxygenation. 2. Incorrect: The client will need to have the machine after surgery. 3. Incorrect: The best response is for the nurse to recommend that the client bring the machine. 4. Incorrect: The nurse can answer this question.

After reviewing the client assignments, the LPN/VN tells the RN the assignment is very unfair and requests that some of the clients be redistributed to the other staff. What should the RN do first? 1. Ask the LPN/VN how the client assignment should be adjusted. 2. Assign one of the LPN/VN's clients to another nurse. 3. Encourage the LPN/VN to use teamwork skills in caring for the clients. 4. Develop a strategic plan to assist with client assignments.

1. Correct: Explore her concerns; this is most therapeutic and helpful response. Finding out what are LPN/VN's concerns first will help the RN address the LPN/VN's request and build trust in the healthcare team relationship. 2. Incorrect: This statement does not help the RN understand the LPN/VN's concern about the assignment, an negates the confidence in the LPN/VN's abilities and skills. 3. Incorrect: This answer does not acknowledge the LPN/VN's concern. 4. Incorrect: This action will not help address the LPN/VN's immediate concern with the assignment and makes resolution of the issue much more complicated than it should be.

The home health nurse is assessing a client whose spouse died in a motor vehicle accident 6 months ago. The client says, "I feel all alone now". Which response by the nurse is therapeutic? 1. "You are feeling all alone." 2. "Why do you say you are lonely?" 3. "Your feelings of loneliness will decrease." 4. "I know other people who lost someone feel this way."

1. Correct: The nurse is utilizing the therapeutic communication technique of restating. The central statement by the client is restated. The client is able to identify that their thoughts or feelings have been received correctly. This also allows the client to clarify their thoughts or feelings if needed. 2. Incorrect: The nurse is requesting an explanation from the client. This is a nontherapeutic communication technique which can cause the client to have to defend their behavior or feelings. 3. Incorrect: The nurse is conveying to the client that their feelings of loneliness will decrease. This response is nontherapeutic communication technique of giving reassurance. The nurse is downplaying the client's feelings of loneliness. 4. Incorrect: The client is expressing their loneliness of losing a spouse. When the nurse states others have experienced loss, the nurse is utilizing the nontherapeutic communication technique of belittling feelings. This response causes the client to question their loneliness.

A client diagnosed with pancreatic cancer is being discharged home to live with an adult child. What action should the nurse take to promote continuity of care? 1. Identify community services available for the client and family. 2. Refer client for hospice care. 3. Advise family that client would benefit more from nursing home placement. 4. Make arrangements for around the clock home health aides.

1. Correct: The nurse promotes continuity of care at discharge by providing a smooth transition from one level of care to another. The nurse should include in the discharge plan appropriate community support services available to the client and family so that they can obtain support as needed. 2. Incorrect: This may be premature at this point. Hospice referral is provided when any person with a life threatening illness, which measures life in months rather than years, qualifies for hospice care. 3. Incorrect: It is not appropriate for the nurse to impose personal opinions about what is best for the client. 4. Incorrect: This may be premature at this point. Further assessment is needed and can be provided as the cancer progresses.

Which postpartum client should the nurse assign to a private room? 1. Has antibodies for Hepatitis C. 2. Is rubella non-immune. 3. Is rubella immune. 4. Has lupus antibodies.

1. Correct: This client should be in a private room for her protection and the protection of other postpartum women. The presence of antibodies for Hepatitis C indicates HCV infection and possibly impaired immune function due to liver damage. In addition, Hepatitis C is transmitted by contact with body fluids and it is likely that lochia will be found on toilet surfaces. It is also common for postpartum women to have some kind of wound (perineal laceration or episiotomy) and they will be at increased risk of HCV contaminated lochia coming into contact with their wound. 2. Incorrect: Rubella non-immunity carries risks only to an unborn fetus. If a non-immune pregnant woman contracts rubella during the first trimester, it can result in serious complications in the fetus. Being rubella non-immune is not of concern when making room assignments for postpartum clients. Being non-immune to rubella does not make this client a risk to other postpartum clients. 3. Incorrect: Rubella-immune woman has no risks. Being rubella immune indicates that the client has developed antibodies in response to a previous rubella infection or immunization. Rubella immunity is desired for pregnant women or those planning to become pregnant. This will help to prevent a rubella infection during pregnancy which could cause birth defects. Being rubella immune would not prevent this client from being able to be placed in the room with other postpartal clients. 4. Incorrect: The woman with lupus antibodies is not at increased risk for infection to herself or to others. The pregnant woman with lupus antibodies requires more vigilant monitoring of the fetus because they may increase the risks of neonatal lupus syndrome, certain heart defects, and miscarriage or pre-term birth. However, these antibodies in the postpartal client does not pose any risks to other postpartal clients, so this would not be a factor when making room assignments.

A client was admitted to CCU with a diagnosis of acute coronary syndrome. Continuous cardiac monitoring has been implemented. Which assessment finding by the nurse is most significant? 1. Ventricular fibrillation 2. Ventricular tachycardia 3. 2nd degree AV block 4. Atrial fibrillation

1. Correct: V-fib is the most common lethal dysrhythmia in the initial period following a myocardial infarction. 2. Incorrect: V-tach is significant as it may occur prior to V-Fib. However, V-fib is most significant. 3. Incorrect: The client will still have a cardiac output in second degree heart block. There is no cardiac output with V-fib. The most lethal is V-fib. 4. Incorrect: Atrial fibrillation involves chaotic contractions of the atria, but there is a cardiac output. It is not life-threatening.

The nurse is monitoring the infection risk in a client that is to begin chemotherapy. Which activity should alert the nurse that the client is at a higher risk for infection? Select all that apply 1. Enjoys getting manicures and pedicures every two weeks. 2. Loves to go with the children to the local water park. 3. Relaxes in hot tubs when traveling. 4. Selects steamed vegetables as part of routine dietary intake. 5. Prefers to go barefooted when at home. 6. Keeps cats in the home and cleans the litter boxes once a week.

1., 2., 3., 5., & 6. Correct: Infection is one of the most common life-threatening complications associated with cancer and chemotherapy. You know that both the cancer and chemotherapy weakens the immune system. Therefore, clients on chemotherapy should be familiar with activities that should be avoided due to the risk of infection with the immunosuppressed state. There are several things that are known to increase the risk of infection in these clients. Did you pick up on these? Well, let's look at a few of these. Clients on chemo should not get manicures or pedicures at salons or spas and should avoid having false nails or nail tips applied. There is too great a risk of contamination at the public salons, so clients are encourage to use their own personal and well-cleaned tools for nail care at home. Another source of bacterial contamination is public water parks. Although these parks take measures to reduce the risk of infection to the general public, the risk is too great for a client on chemo. Swimming can result in accidental ingestion of water which increases the risk of cryptosporidium or other waterborne pathogens. Same thing applies to hot tubs. ​So why is going barefoot at home such a big deal? Well, this increases the risk of cuts, scrapes, or other injury that would increase the portal for infectious agents to enter. In addition, the exposure to potential infectious agents is greater. The oncologist may direct the client in the best way to deal with this client having cats in the home and cleaning the litter box due to the risk of exposure to bacteria and parasites. If allowed to clean the litter box, latex or rubber gloves, along with a mask over the nose and mouth is generally recommended to reduce the risk of infection. In addition, the client should be instructed to thoroughly wash the hands with soap and water after cleaning the litter box or after touching the cats. 4. Incorrect: Although the intake of fresh fruits and vegetables has been controversial, most agree that if washed properly, even fresh fruits and vegetables can be consumed. However, the oncologist should be the one to approve the dietary intake of these. Here, we have vegetables that have not only been washed, but steamed as well. These should be safe for consumption for clients on chemo.

A pediatric nurse is providing anticipatory guidance to a group of parents who have children nearing the age of 1 year old. What milestones should the nurse teach the parents to expect to see in their 1 year old child? Select all that apply 1. Gets to a standing position without help. 2. Puts out arm or leg to help with dressing. 3. Able to say several single words. 4. Pulls toys while walking. 5. Builds a tower of 4 blocks.

1., & 2. Correct: A 1 year old should be able to get to a standing position without help. May stand alone. Can assist in getting dressed by putting out arm or leg. 3. Incorrect: Children at 18 months are able to say several single words. 4. Incorrect: Children at 18 months are able to pull toys while walking. 5. Incorrect: Children at 2 years of age can build a tower of 4 or more blocks.

The nurse is teaching a community education course regarding complementary and/or alternative therapies. Which therapies would the nurse include in the course as complementary and/or alternative therapies? Select all that apply 1. Acupuncture 2. Yoga 3. Tai chi 4. Reiki 5. Zumba

1., 2, 3, & 4. Correct: All are considered complementary and/or alternative therapies. Acupuncture involves stimulating specific points on the body. This is most often done by inserting thin needles through the skin, to cause a change in the physical functions of the body. Research has shown that acupuncture reduces nausea and vomiting after surgery and chemotherapy. It can also relieve pain. The practice of yoga makes the body strong and flexible, and improves the functioning of the respiratory, circulatory, digestive, and hormonal systems. Yoga brings about emotional stability and clarity of mind. Tai chi is an ancient Chinese discipline involving a continuous series of controlled usually slow movements designed to improve physical and mental well-being. Reiki is a healing technique based on the principle that the therapist can channel energy into the client by means of touch, to activate the natural healing processes of the body and restore physical and emotional well-being. 5. Incorrect: Zumba is a type of dance exercise and is not considered a form of alternative therapy.

The home care nurse is caring for an elderly client status post total hip replacement and a history of cirrhosis. Which statements by the client's spouse indicates that teaching regarding pain management has been successful? Select all that apply 1. "If the pain increases, I must let the nurse know immediately." 2. "I should have my spouse try the breathing exercises to help control pain." 3. "This narcotic causes very deep sleep, which is what my spouse needs." 4. "If constipation is a problem, increased fluids will help." 5. "My spouse can have one glass of wine to help promote pain relief."

1., 2., & 4. Correct: These are correct responses by the spouse. Increased pain may indicate something else is going on. Breathing exercises would be an excellent non-pharmacological intervention. Increasing fluid is an appropriate intervention for constipation. Narcotics place the client at risk for constipation. 3. Incorrect: The nurse should know that goal of pain relief and administration of narcotics is to use the smallest dose possible to relieve the pain. Narcotics are not used to put clients into deep sleep. Also, clients with liver failure have less tolerance to narcotics because they cannot mobilize drugs like a normal person. Narcotics must be given cautiously to clients with liver disease and the elderly. 5. Incorrect: Alcohol should not be taken when a client has cirrhosis. Alcohol is hepatotoxic to the client with liver disease.

The nurse suspects a client admitted with myasthenia gravis is going into a cholinergic crisis. Which signs and symptoms would validate the nurse's suspicions? Select all that apply 1. Diarrhea 2. Increased urination 3. Dilated pupils 4. Tachycardia 5. Nausea and vomiting

1., 2., & 5. Correct: The signs of cholinergic crisis include Diarrhea and abdominal cramping, Urination increased, Miosis (pinpoint pupils), Bradycardia, Emesis (nausea and vomiting), Lacrimation, Lethargy, Salivation. Remember this: DUMBELLS as a mnemonic to help you recall these signs and symptoms. 3. Incorrect: Pinpoint pupils rather than dilated pupils will occur. 4. Incorrect: Tachycardia is not a sign of cholinergic crisis. Bradycardia will occur.

The nurse is educating a group of sexually active teenagers about Chlamydia. What should the nurse teach these clients to prevent them from acquiring or transmitting this disease ? Select all that apply 1. Use a latex condom when having sex to protect against Chlamydia. 2. Seek the advice of a primary healthcare provider if there is vaginal discharge or burning on urination. 3. Suggest that the teens be screened for Chlamydia. 4. Reassure the teens that if they have no symptoms, they have no disease. 5. Take prescribed medication if diagnosed with Chlamydia, and repeat screening in three months.

1., 2., 3. & 5. Correct: Consistent use of latex condoms protects against STIs. Although chlamydia may have no symptoms, burning and discharge should be reported for further evaluation. It is recommended that all sexually active young women less than 25 years of age be screened for chlamydia on an annual basis. Medication should be taken as prescribed, and rescreening should occur in 3 months to make sure that there is no more disease present. 4. Incorrect: Chlamydia does not always produce visible symptoms, and, if left untreated, can lead to pelvic inflammatory disease (PID). False security may lead to unsafe sex practices.

What action should the nurse take after mistakenly administering the wrong medication? Select all that apply 1. Notify the nursing supervisor. 2. Inform the primary healthcare provider. 3. Complete an incident (variance) report. 4. Document client assessment and response to medication. 5. Document medication error and incident (variance) report in nurse's notes.

1., 2., 3., & 4. Correct: Nurses must immediately report all client care issues, concerns or problems to the supervising nurse, the primary healthcare provider and/or the performance improvement or risk management department. A written report of the incident is completed by the nurse and turned into the appropriate person (generally the performance improvement department). Documentation of what occurred, and the client's assessment is required in the nurse's notes. 5. Incorrect: Do not document that an error was made or that an incident (variance) report was completed. Document what medication was given, the client's assessment, the notification of the nursing supervisor, and primary healthcare provider, and any prescriptions received.

A nurse manager has several issues regarding staff maintaining proper infection control while caring for clients. What actions should the manager take regarding this issue? Select all that apply 1. Place colorful posters regarding infection control in conspicuous places on unit. 2. Monitor staff providing client care for the use of appropriate infection control. 3. Give staff a written test on proper infection control. 4. Have all staff read agency policy and procedures regarding infection control. 5. Dock pay of staff who do not maintain proper infection control. 6. Provide mandatory in-service sessions on infection control for every shift.

1., 2., 3., 4., & 6. Correct. Each of these actions can be taken by the nurse manager. The staff needs further education, reminders, and follow-up observation. Posters are great reminders of concepts. All nurses need to supervise those under their direction. Testing can be done as pretest or post test along with in service education. Staff development or in service sessions are required by Joint Commission on Accreditation of Healthcare Organizations (JCHO) on infection control. 5. Incorrect. This is not the best solution, because most people want to do what is right. Education should be tried first, then documentation of the infractions. You must support, supervise and educate!

The nurse is caring for a client diagnosed with myasthemia gravis. What assessment finding would the nurse expect to see in this client? Select all that apply 1. Difficulty chewing 2. Drooping eyelids 3. Facial paralysis 4. Hoarseness 5. Limb rigidity 6. Trouble talking

1., 2., 3., 4., & 6. Correct: The main symptom of MG is weakness in the voluntary skeletal muscles, which are muscles under a person's control. The failure of muscles to contract normally occurs because they can't respond to nerve impulses. Symptoms can include difficulty chewing, drooping of eyelids, facial paralysis, hoarseness, and trouble talking speaking. 5. Incorrect: Limb rigidity is a sign of Parkinson's disease.

The nurse suspects a client admitted with myasthenia gravis is going into a cholinergic crisis. Which signs and symptoms would validate the nurse's suspicions? Select all that apply 1. Abdominal cramping 2. Lethargy 3. Salivation 4. Hypertension 5. Lacrimation 6. Miosis

1., 2., 3., 5., & 6. Correct or True: The signs of cholinergic crisis include Diarrhea and abdominal cramping, Urination increased, Miosis (pinpoint pupils), Bradycardia, Emesis (nausea and vomiting), Lacrimation, Lethargy, Salivation. Remember this: DUMBELLS as a mnemonic to help you recall these signs and symptoms. 4. Incorrect or False: Hypertension is not a sign of cholinergic crisis. Muscles get weaker so BP would go down.

A nurse is educating a group of community citizens about risk factors for developing peripheral neuropathy. Which risk factors should the nurse include? Select all that apply 1. Uncontrolled diabetes 2. Alcohol abuse 3. Vitamin A deficiency 4. Rheumatoid arthritis 5. Varicella-zoster virus

1., 2., 4. & 5. Correct: All are risk factors for peripheral neuropathy. 3. Incorrect: Vitamin A deficiency can result from inadequate intake, fat malabsorption, or liver disorders. Deficiency impairs immunity and hematopoiesis and causes rashes and typical ocular effects (e.g., xerophthalmia, night blindness). Vitamin B deficiency can cause peripheral neuropathy.

What lab values should the nurse monitor when caring for a client diagnosed with acute leukemia? Select all that apply 1. Hemoglobin 2. Hematocrit 3. Lactate dehydrogenase (LDH) 4. Platelets 5. White blood cells 6. Metanephrine

1., 2., 4., & 5 Correct: The client with acute leukemia usually has a decreased hemoglobin and hematocrit level, a low platelet count, and an abnormal white blood cell count. 3. Incorrect: If lymphoma has been diagnosed, the lactate dehydrogenase (LDH) level may be checked. LDH levels are often increased in patients with lymphomas. LDH is not a lab test monitored in the client with acute leukemia. 6. Incorrect: Plasma free metanephrine is up to 99% sensitive for diagnosing pheochromocytoma. It measures circulating epinephrine and norepinephrine levels. Grossly elevated plasma norepinephrine renders the diagnosis of pheochromocytoma highly probable.

What food should the nurse include when teaching an older adult about increasing vitamin B12 intake? Select all that apply 1. Calf liver 2. Feta cheese 3. Fresh spinach 4. Shrimp 5. Tuna 6. Tofu

1., 2., 4., & 5. Correct: A serving of just one ounce of beef liver contains well over the amount of B12 that the average person needs for the day. Feta cheese contains a good amount of vitamin B12 as well as several other important nutrients such as calcium and vitamin B2. Shrimp contain about 80 percent of the daily value needed of vitamin B12. Tuna either canned or grilled has a lot of vitamin B12. In fact, just one three ounce serving offers an entire day's worth of vitamin B12. 3. Incorrect: Fresh spinach does not have any vitamin B12. 6. Incorrect: Tofu is low-carb, dairy-free, gluten-free, cholesterol-free, and vegan, so it is popular with people who have specialized diets. However, vitamin B12 is not in tofu.

A nurse is caring for a client diagnosed with the ebola virus who is experiencing vomiting and diarrhea. What personal protective equipment should be worn by the nurse while providing care to this client? Select all that apply 1. Single use impermeable gown 2. Powered Air Purifying Respirator (PAPR) or N95 respirator 3. One pair of sterile gloves 4. Single use boot covers 5. Single use apron

1., 2., 4., & 5. Correct: The nurse should wear a single use (disposable) impermeable gown OR a single use impermeable coverall. Either a PAPR or a disposable, NIOSH-certified N95 respirator should be worn to reduce the risk of contamination in the case of an emergency situation where a potentially aerosol-generating procedure would be performed. The PAPR reduces the risk of self-contamination while providing client care, but the N95 respirator is less bulky. If the N95 respirator is selected for use, nurses should be extremely careful to make sure that they do not accidentally touch their faces under the face shield during client care. Disposable boot covers should be worn and should extend to at least mid-calf. Some agencies may add the single use shoe covers over the boot covers to reduce the risk of contaminating the underlying shoes. If clients with Ebola are vomiting or have diarrhea, a single use (disposable) apron should be worn over the gown to cover the torso to mid-calf. This will provide additional protection to reduce the risk of contaminating the gown (or coveralls) by the infectious body fluids and also provides a way to rapidly remove a soiled outer layer if contamination occurs on the apron. 3. Incorrect: Sterile gloves are not required, but two pairs, instead of one pair, of gloves should be worn so that a contaminated outer glove can be safely removed when providing client care or safely removed without self-contamination when removing the PPE. These gloves should at the very least have extended cuffs.

What symptoms of meningeal irritation would the nurse anticipate when performing an assessment on a newly admitted client with a diagnosis of bacterial meningitis? Select all that apply 1. Positive Kernig's sign 2. Positive Brudzinski's sign 3. Absence of Babinski's reflex 4. Photophobia 5. Severe headache 6. Nuchal rigidity

1., 2., 4., 5., & 6 Correct: Brudzinski's sign is the involuntary lifting of the legs when the neck is passively flexed (head is lifted off the examining surface). Kernig's sign is positive when the thigh is bent at the hip and knee at 90 degree angles and attempts to extend the knee are painful, resulting in resistance. Both of these signs are thought to indicate meningeal irritation. These seem to be caused when the motor roots become irritated as they pass through inflamed meninges, and the roots are brought under tension. Photophobia (sensitivity to bright light), severe, unrelenting headache, and nuchal rigidity (stiff neck) are all believed to be due to irritation of the meninges. 3. Incorrect: Babinski reflex is a normal reflex in infants up to age 2, but is a pathological reaction in adults. It is often indicative of severe damage to the central nervous system, including meningeal irritation.

Which signs and symptoms does the nurse expect to see in a client admitted to the medical unit with Parkinson's disease? Select all that apply 1. Blank affect. 2. Decreased ability to swing arms. 3. Waddling gait. 4. Walking on toes. 5. Pill-rolling tremor. 6. Stiff muscles.

1., 2., 5., & 6. Correct: Classic characteristics of Parkinson's disease include a blank facial expression, forward tilt in the posture, slow/slurred speech, tremor, and a short shuffling gait. These symptoms also are manifested by a decreased ability to swing the arms and stiff muscles. 3. Incorrect: This is a sign of Duchenne Muscular Dystrophy. The client with Parkinson's disease has a shuffling gait. 4. Incorrect: This is a sign of Duchenne Muscular Dystrophy. The client with Parkinson's disease has a shuffling gait.

A client requires external radiation therapy. The nurse knows external radiation may cause which problems? Select all that apply 1. Pancytopenia 2. Leukocytosis 3. Erythema 4. Fever 5. Fatigue

1., 3. & 5. Correct: Effects of radiation therapy include, but are not limited to pancytopenia (marked decrease in the number of RBCs, WBCs and platelets), erythema (redness of the skin), and fatigue. 2. Incorrect: Leukocytosis is an increase in WBCs. External radiation causes pancytopenia which is a decrease in the number of blood cells including WBCs. 4. Incorrect: Fever is not typically seen with external radiation.

Which discharge instruction should the nurse implement for a client diagnosed with insomnia? 1. Eliminate chocolate in the evening. 2. Drink a glass of red wine 1 hour prior to bedtime. 3. Perform progressive relaxation techniques at bedtime. 4. Take acetaminophen/diphenhydramine 2 tablets at bedtime. 5. Leisurely walk 3 hours prior to bedtime. 6. Increase the air flow on the continuous positive airway pressure (CPAP) machine.

1., 3., & 5. Correct: Consuming chocolate in the evening may cause insomnia. Chocolate contains caffeine and xanthines which are stimulates. The chemicals will suppress melatonin and increase the time to fall asleep. Progressive relaxation techniques are recommended to reduce insomnia. This exercise is a systematic relaxation and tensing of the muscle groups of the body. Insomnia is reduced by increasing muscle relaxation and decreasing the stress level of the client. Nonstrenuous exercises such as a leisure walk performed within 3 hours of bedtime promotes the reduction of the client's stress level. 2. Incorrect: Consuming alcohol prior to bedtime is not recommended. Alcohol consumption increases the start of sleep but reduces rapid eye movement (REM) sleep. The side effect of the alcohol may also cause the client to awaken during night and have difficulty returning to sleep. 4. Incorrect: Diphenhydramine is not recommended for insomnia. The action of the diphenhydramine may cause the client to feel drowsy but provides only temporary increase in quantity of sleep. The hypnotic effect of diphenhydramine will cause client to experience decrease energy levels the next morning. 6. Incorrect: A CPAP is prescribed for a client with obstructive sleep apnea not insomnia. The CPAP machine delivers a constant air pressure to the lungs. The constant air flow will keep the airway open during sleep.

What nursing interventions should the nurse include when planning care for a client admitted with Guillain-Barre' Syndrome? Select all that apply 1. Monitor for contractures. 2. Place prone for 30 minutes, 4 times per day. 3. Provide therapeutic massage for pain relief. 4. Teach range of motion exercises. 5. Provide high protein meals 3 times a day. 6. Refer to physical therapist.

1., 3., 4., & 6. Correct: This client will have progressive weakness and paralysis. Contractures and pressure ulcers need to be prevented through ROM exercises and frequent turning. Muscle spasms and pain can be relieved by therapeutic massage, imagery, diversion, and pain medication. 2. Incorrect: The client will need to be repositioned every 2 hours to prevent pressure sores and pneumonia and atelectasis. Elevate the head of the bed to help with lung expansion. Prone will interfere with lung expansion ability. 5. Incorrect: Encourage small, but frequent meals that are both well-balanced and nourishing.

Which tasks are most appropriate for the hospice nurse to delegate to an unlicensed assistive personnel (UAP)? Select all that apply 1. Bathe the client. 2. Provide spiritual support 3. Listen to the client reminisce. 4. Administer routine medications. 5. Weigh the client. 6. Take vital signs

1., 3., 5., & 6. Correct: The UAP can bathe, listen to the client remininsce, weigh, and take the vital signs. These are within the scope of practice of the UAP. These assignments are routine and revolve around activities of daily living. 2. Incorrect: The task of providing spiritual support could best be delegated to the pastor or chaplain. 4. Incorrect: The nurse can not delegate routine medication administration to the UAP. This is not within the UAPs scope of practice. This is an LPN or RN responsibility.

The nurse is teaching a client who is at risk for developing a stroke. What primary prevention strategies should the nurse include? Select all that apply 1. Promote a diet rich in fruits and vegetables. 2. Provide instruction on benefits of carotid endarterectomy. 3. Limit sodium intake to 2 grams/day. 4. Engage in low intensity exercise once a week. 5. Avoid tobacco products. 6. Decrease alcohol consumption to two drinks per day.

1., 3., 5., & 6. Correct: These strategies are considered primary prevention strategies that can decrease the risk of developing a stroke. 2. Incorrect: This would be considered secondary prevention: early diagnosis and treatment to prevent stroke. 4. Incorrect: The client needs 3-4 sessions per week of moderate-vigorous intensity aerobic physical exercise to reduce stroke risk factors. Session should last an average of 40 minutes. Moderate intensity exercise is typically defined as sufficient to break a sweat or noticeably raise heart rate (e.g. walking briskly, using an exercise bicycle). Vigorous intensity exercise includes activities such as jogging.

Which cranial nerves should the nurse assess in a client diagnosed with Guillain-Barre' Syndrome? Select all that apply 1. Vagus 2. Olfactory 3. Vestibulocochlear 4. Facial 5. Trigeminal 6. Oculomotor

1., 4., 5., & 6. Correct: Cranial nerves III - VII and IX-XII may be affected by Guillain-Barre' Syndrome and should therefore be monitored by the nurse. Facial drooping, diplopias, dysarthria, dysphagia, ophthalmoplegia, and pupillary disturbances are commonly seen. 2. Incorrect: The first cranial nerve is not generally affected by this disease. 3. Incorrect: Hearing is controlled by the 8th cranial nerve which is almost never involved.

A 154 pound (70 kg) client is admitted to the burn unit with second and third degree burns covering 50% total body surface area. Normal Saline IV fluid resuscitation is ordered at 4 ml/kg per percentage of total body surface area burned over the first 24 hours. How much fluid does the nurse calculate the client will receive in 24 hours? Provide your answer using whole numbers.

154 pounds/2.2 kg = 70 kg 4 mL x 70 kg = 280 280 mL x 50 tbsa = 14,000 mL in the first 24 hours

A client weighing 140 pounds (63.64 kg) has been admitted to the telemetry unit with a diagnosis of Class III pulmonary hypertension. The primary healthcare provider prescribes digoxin. How many micrograms should the nurse administer now? Round to the whole number.

140 pounds / 2.2 kg = 63.6363636 63.6363636 x 15 micrograms = 954.545455 954.545455 / 2 = 477.272727 or 477 micrograms now The following 2 doses would be 239 micrograms.

What interventions should the nurse include in the care plan of a client admitted with Guillain-Barre syndrome? 1. Assess for descending paralysis. 2. Keep a sterile tracheostomy at the bedside. 3. Monitor for heart rate above 120/min. 4. Maintain in side-lying, supine position. 5. Have client perform active range of motion (ROM) every 2 hours while awake.

2. & 3. Correct or True: This client is at risk for respiratory paralysis as the disease progresses. An emergency tracheostomy may need to be performed so the nurse should watch out for imminent signs of respiratory failure. Signs include heart rate that is more than 120 bpm or lower than 70 bpm and respiratory rate of more than 30 bpm. The nurse should assess for signs of respiratory distress and prepare for intubation if needed. 1. Incorrect or False: Ascending paralysis should be assessed for with this disease. Paralysis begins in the lower extremities and moves upward. 4. Incorrect or False: The client should be assisted to a position with head of bed elevated for full chest excursion. 5. Incorrect or False: The nurse should perform passive range of motion exercises. Active exercise should be avoided during the acute phase as the client is easily fatigued and muscles are weak. Passive ROM stimulates circulation, improves muscle tone and increases joint mobilization.

A distraught client arrives at a mental health crisis center following a house fire that also took the life of a young family member. The nurse knows what action is most important when initiating crisis intervention for this client? 1. Assist the client to verbalize feelings of grief. 2. Assess client for any suicidal behaviors. 3. Admit client to general mental health unit. 4. Assign client to a grief counseling group.

2. Correct: Client safety is always the nurse's priority concern where no other life threatening issues exist. A distraught client in crisis from such overwhelming events does not always think or act clearly. The loss of home combined with the death of a family member places the client at potential risk for suicide. Because the client has presented to the mental health crisis center, the nurse must assume the worst and assess for unexpected responses. 1. Incorrect: While it is true that encouraging the client to verbalize feelings is therapeutic in a crisis, that is not the most important initial action by the nurse at this time. Recall the nursing process when considering an irrational action. 3. Incorrect: Arriving at a mental health crisis center does not automatically require admission to the hospital. This client is overwhelmed by circumstances which include the death of a family member; however, ideally the client may respond to counselling or medications without the need for inpatient care. 4. Incorrect: Following evaluation by a primary healthcare provider, this client will definitely receive counseling, perhaps both individualized and in a support group for those under extreme duress. However, this is not the initial concern for the nurse.

A client enters the post-anesthesia care unit with a three way indwelling urinary catheter that has a continuous irrigation of normal saline infusing. The urine in the indwelling urinary catheter bag, is dark red. Which action should the nurse take first? 1. Chart the drainage color and amount. 2. Increase the flow rate of the irrigation solution until the urine is a light pink. 3. Notify the primary healthcare provider of the dark red drainage. 4. Pull traction on the indwelling tubing and tape the indwelling tubing to the client's leg.

2. Correct: Continuous bladder irrigation is used following surgery to ensure that the bladder remains clear of blood clots.The nurse would need to increase the irrigation rate until the urine becomes light pink. 1. Incorrect: If the urine is not diluted, the client could form clots in the urine that could obstruct the urine flow. Charting the drainage color and amount would not address the issue. 3. Incorrect: Dark red color to the urine would warrant an increase in irrigation. There is no need to call the primary healthcare provider. If the color of urine doesn't clear or the vital sign show signs of shock (increased heart rate and decreased blood pressure) then notifying the primary healthcare provider would be needed. 4. Incorrect: This is the intervention that would be carried out if the client is hemorrhaging from the prostate. The balloon on the catheter would be used to apply pressure to the prostate and decrease bleeding. If there was more evidence of hemorrhage such as a decrease in blood pressure or increase in heart rate this type traction would be initiated. There is not enough evidence of hemorrhage at this point to initiate traction, therefore, more assessments should be performed.

An elderly client is admitted to the floor with vomiting and diarrhea for three days. The client is receiving IV fluids at 200 mL/hr via pump. What would be the priority nursing action? 1. Obtaining Intake and Output 2. Frequent lung assessments 3. Vital signs every shift 4. Monitoring the IV site for infiltration

2. Correct: IV fluids at 200 mL/hr is a rapid infusion rate. The elderly adult is at risk for circulatory overload and should be closely monitored during rapid infusion rates. Lung assessments are important in detecting fluid overload. The client may experience shortness of breath and moist crackles on auscultation. 1. Incorrect: I and O are important, but less priority than lung assessment in the elderly client. 3. Incorrect: Vital signs should probably be more frequent than every shift on the elderly client with dehydration. 4. Incorrect: The IV site should be monitored for infiltration, but will not be priority over lung assessment in the elderly client.

A resident who shares a semi-private room with a terminally ill resident in a long- term facility becomes aware of the death of a prior roommate. The resident states "We were just talking this morning." Which communication response would the nurse initiate? 1. "I think you will feel better later." 2. "You were talking this morning?" 3. "Now you know I cannot respond to you." 4. "Why did you say that you talked this morning?"

2. Correct: Other residents in the long-term facility may become aware of death in the facility. The nurse should initiate comfort interventions for these residents. The nurse is utilizing the therapeutic communication technique of restating. The resident is exhibiting the denial stage of grief. 1. Incorrect: The other residents in a long-term facility may become aware of the death of another resident. This answer by the nurse is giving reassurance, a non-therapeutic communication technique, and will hinder the resident from sharing their feelings. 3. Incorrect: Due to the proximity of the residents in a long-term facility, many of the residents become aware of a death of another resident. The nurse is stopping the resident from further communication by utilizing the non-therapeutic technique of rejecting. The nurse indicates that there will be no further discussion. 4. Incorrect: By using the word why, the nurse is requesting a reason for the resident's comment. This is the non-therapeutic technique of requesting an explanation. The client is placed in a position of defending the comments.

The nurse is teaching a client about the use of a cane. Which is the correct cane technique? 1. Place the cane on weaker side of the body to support the weaker leg. Using the cane for support, the client should step forward with strong leg, and then move the weaker leg and cane forward to the strong leg. 2. Place the cane on the stronger side of the body. The cane is placed forward 6 to 10 inches while the client advances the weak leg at the same time. 3. Place cane on weaker side of body. The cane is placed forward 6 to 10 inches while the client advances weaker leg to the cane. 4. Place cane on stronger side of body to help support weaker leg. Using cane for support, step forward with the strong leg and then move the weaker leg and the cane forward to the strong leg.

2. Correct: Place the cane on the stronger side of the body. The cane is placed forward 6 to 10 inches while the client advances the weak leg at the same time. The body weight is divided between the strong leg and the cane. 1. Incorrect: The cane should be on the stronger side of the body to create a wider base for balance as the client advances the strong leg and must use the weaker leg for support with the cane. If the cane is placed on the weaker side of the body, this would create a narrower base for support and balance and increase the risk of falling. 3. Incorrect: The cane should be on the stronger side of the body to create a wider base for balance as the client advances the strong leg and must use the weaker leg for support with the cane. If the cane is placed on the weaker side of the body, this would create a narrower base for support and balance and increase the risk of falling. 4. Incorrect: The cane should be on the stronger side of the body to create a wider base for balance as the client advances the strong leg and must use the weaker leg for support with the cane. If the cane is placed on the weaker side of the body, this would create a narrower base for support and balance and increase the risk of falling.

A client has been admitted with a diagnosis of sepsis and two sets of blood cultures have been ordered. When the nurse explains the procedure, the client asks the purpose of drawing blood from two different veins at two different times. What is the best response by the nurse? 1. "If we don't get enough blood the first time, we can obtain more." 2. "We want to be sure to get samples of all organisms in your blood." 3. "We have to be certain none of the samples have been contaminated." 4. "It's important not to get too much blood from the arm all at once."

2. Correct: Sepsis, sometimes referred to as septicemia, is a serious infection in the bloodstream which can be caused by bacteria, parasites, fungi or even a virus. Blood cultures provide vital information regarding what organisms are present in the blood so the proper medication can be ordered. Standard procedure is to obtain two separate blood samples from two separate veins approximately 30 minutes apart to be certain of obtaining samples of all organisms present. 1. Incorrect: The purpose of obtaining several blood samples has nothing to do with getting "enough blood". The purpose is to obtain an accurate cross-section of all organisms present in the blood. 3. Incorrect: Blood samples are generally not contaminated when drawn from the client, but rather in the processing phase of analysis. 4. Incorrect: Blood cultures are obtained in glass bottles and the amount required is quite small.

A client with severe depression and a previous history of attempted suicide has been receiving inpatient therapy for months. The nurse notes at breakfast the client is showered, in clean clothes with hair combed. What response by the nurse is most therapeutic at this time? 1. "You look great today, so you must be feeling better." 2. "I see you are wearing a bright blue sweater today." 3. "Has something changed in your life this morning?" 4. "Today must be a very special occasion for you."

2. Correct: The nurse is focusing on the 'here and now' by acknowledging the client's changed appearance without adding any personal comments. This response avoids typical communication blocks such as "you look great", which seems positive but is considered a judgment by the nurse. The nurse's broad statement shares an observation in an open-ended manner. 1. Incorrect: This remark provides false reassurance to the client by assuming change in visual appearance must be based on emotional improvement. This is very misleading and non-therapeutic for the client. 3. Incorrect: Questions can be positive communication tools if presented in an open-ended format. Asking about life changes in this manner could be answered yes or no, which indicates a closed-ended question. A sudden improvement in appearance may be one of the warning signs of suicidal ideations, but the nurse's inquiry would not provide the information needed. 4. Incorrect: The nurse is inferring the client needs a special occasion in order to be clean and well-kempt. Such a comment is presumptuous and judgmental.

Following escharotomy of a circumferential burn to the arm, which assessment is the best indicator when evaluating the effectiveness of this procedure? 1. Decreased pain in the extremity 2. Prompt capillary refill < 2 seconds after blanching 3. Bleeding at the site of the incision 4. Ability of the client to wiggle his/her fingers

2. Correct: The objective of creating an incision through the eschar is to relieve the pressure and restore circulation. If nail beds blanch and refill promptly, blood is flowing into the limb. 1. Incorrect: Decreased pain is not an indicator of circulation to a limb. It's a good thing, but is not a definitive evaluation of circulation. 3. Incorrect: Bleeding would indicate that circulation was improved in the incision area, but to assure improved circulation of the total arm, capillary refill is the best assessment. 4. Incorrect: Movement (motor) is a neurological check, and the right answer will involve a circulatory (vascular) check!

Which action by two unlicensed nursing personnel (UAPs), while moving the client back up in bed, would require intervention by the nurse? 1. Lowers the side rails closest to them. 2. Places hands under client's axilla. 3. Lowers the head of bed. 4. Raises the height of the bed.

2. Correct: This action is not appropriate and requires intervention by the nurse. This could damage the brachial plexus nerves under the axilla. Use a draw sheet to prevent this from occurring. 1. Incorrect: This is a correct action. The UAPs will need to lower the side rails closest to them to safely move the client up in bed. Not lowering the rails could injury the UAPs back. 3. Incorrect: This action is correct. Moving the client upward with the head of the bed raised works against gravity, requires more force and can cause back strain. 4. Incorrect: This action is appropriate and would not require intervention by the nurse. Raising the height of the bed brings the client close to the UAPs center of gravity and decreases the chance of back injury.

After completing the initial morning assessment of a client, the nurse notes that a dose of intranasal desmopressin is to be administered. What action is most important for the nurse to take? Exhibit 1. Measure urine osmolality. 2. Hold desmopressin dose. 3. Administer acetaminophen for headache. 4. Instruct client on intranasal administration of desmopressin.

2. Correct: This client is in fluid volume excess, so the desmopressin needs to be held. The nurse needs to discuss the dose with the primary healthcare provider. 1. Incorrect: Urine osmolality can be assessed but it is not the priority. There are lots of data findings indicating that the client is in a fluid volume excess. 3. Incorrect: There is nothing wrong with giving the client acetaminophen for reports of a headache, but it is not the most important action for the nurse to take. 4. Incorrect: Teaching is an important nursing task. However, it is not the most important task now. Teaching can be done later.

A nurse has provided postpartum discharge instructions to a client who had a cesarean section. What statement by the client would indicate to the nurse that further teaching is necessary? 1. "I will relax and contract my pelvic floor muscles 10 times, eight times a day." 2. "Driving is permitted in one week if I am pain free." 3. "Lifting anything heavier than my baby is not advised." 4. "I will not cross my legs while sitting."

2. Correct: This is an incorrect statement. C-sections require a much longer recovery. The client will not have the abdominal muscles to press down on the brake pedal in an emergency. Therefore, new moms who had C-sections should wait until after the three week postpartum appointment to drive. 1. Incorrect: This would be a correct statement. The client should do Kegel exercises to prevent incontinence and strengthen the pelvic muscles. 3. Incorrect: This is a correct statement. Lifting heavy objects can cause bleeding. New moms are not able to lift anything more than the baby's weight. 4 Incorrect: This is a correct statement. Avoid sitting for prolonged periods of time with legs crossed to prevent thrombophlebitis.

A new nurse is anxious about being assigned to a a client with violent episodes. Which statement by the charge nurse would address the new nurse's anxiety? 1. "What you really mean is that you fear a client with violent episodes." 2. "Though it is difficult, the staff needs to remain relaxed, but conscious of the client's violent episodes." 3. "I will instruct the staff to monitor the client's behavior for any signs of violent behavior." 4. "You attended an in-service during orientation on dealing with the client with violent behavior."

2. Correct: This response focuses both on the client's and the staff's response to the client. This is an example of the therapeutic communication of restating. The safety of the client, other clients, and the health care team is also a priority. The aggression by the client may be physical, verbal or both. The nurse should remain calm and firm. This approach will assist the client to return to their pre-crisis state. 1.Incorrect: The charge nurse is concluding the meaning of what the new nurse is saying. This is an example of the nontherapeutic communication of interpreting. The charge nurse stated that the new nurse is scared. This response may block further discussions with the new nurse about nursing care for potentially violent clients. The client, other clients, and the healthcare team's safety can also be placed at risk. 3. Incorrect: The charge nurse is changing the subject by stating that the staff members will be directed to monitor the client for violent behavior. This nontherapeutic communication of introducing an unrelated topic allows the charge nurse to control the direction of the conversation. The new nurse is expressing her concerns about caring for a client with violent episodes. The charge nurse is not focusing on the concerns of the new nurse. 4. Incorrect: The charge nurse responds by stating the new nurse attended an in-service during orientation about dealing with a client with violent behavior. The charge nurse does not identify that the new nurse may be uncomfortable dealing with clients with violent behavior. This is the nontherapeutic communication technique of rejecting. The new nurse may stop sharing with the charge nurse because of concern over further rejection.

A nurse educator has completed an educational program on interpreting arterial blood gases (ABGs). The educator recognizes that education has been successful when a nurse selects which set of ABGs as metabolic acidosis? 1. pH - 7.32, PaCO2 - 48, HCO3 - 23 2. pH - 7.29, PaCO2 - 42, HCO3 - 19 3. pH - 7.5, PaCO2 - 30, HCO3 - 22 4. pH - 7.35, PaCO2 - 35, HCO3 - 26

2. Correct: This set of ABGs reflects the presence of metabolic acidosis. The pH is low (acidic) and the bicarb is low (decreased buffering ability). The carbon dioxide is normal. 1. Incorrect: This is respiratory acidosis. The pH is low (acidic) and the PaCO2 is high. The bicarb is normal. 3. Incorrect: This is respiratory alkalosis. The pH is increased (alkaline) and the PaCO2 is low. The bicarb is normal. 4. Incorrect: This is an example of normal ABGs.

A client is curious about visible appearance changes related to menopause. What menopausal changes, in general, would the nurse explain to the client? 1. Bone loss and fractures. 2. Loss of muscle mass. 3. Improved skin turgor and elasticity. 4. A reduction in waist size.

2. Correct: Visible changes associated with menopause include loss of muscle mass, increased fat tissue leading to thicker waist, dryness of the skin and vagina, hot flashes, sleep abnormalities, and mood changes. 1. Incorrect: Bone loss is dependent on bone mass, weight-bearing exercise, and nutrition. Some bone loss may occur, but may not lead to fractures. 3. Incorrect: A decrease in turgor and elasticity may occur as we grow older. 4. Incorrect: There is increased fat tissue with an increase in waist size.

A client is admitted with a hip fracture after falling. Based on these lab values, what is the nurse's priority nursing intervention? Exhibit Na+ 147 mEq/L (147 mmol/L) Specific gravity 1.030 Hct 55% 1. Provide foods high in iron 2. Increase fluid intake 3. Obtain a urine for culture 4. Measure intake and output

2. Correct: We already know that the question is about what life threatening complication? A pulmonary embolism. And these lab values say that the client is what? Dehydrated! So the only thing that is going to fix that is....... Increasing fluids. 1. Incorrect: This will not prevent pulmonary embolism. The problem is dehydration. Do something to fix the problem. Foods high in iron will not fix the problem. 3. Incorrect: This will not prevent pulmonary embolism. How will obtaining a urine sample for culture fix dehydration? It won't. This client needs to increase fluid intake. 4. Incorrect: We do want to monitor intake and output to see how the client is doing, however, this will not fix the problem. Hydrating the client will help the problem.

Which client is legally able to sign a consent for surgery? Select all that apply 1. An 86 year old client who is disoriented. 2. A 62 year old client who speaks only Spanish. 3. A 41 year old client who just received midazolam. 4. A 17 year old client needing an emergency appendectomy whose parents cannot be contacted. 5. A 44 year old with schizophrenia who is hallucinating.

2., & 4. Correct: The Spanish speaking client should have a trained medical interpreter, either in person,by telephone, or by video conference, but the client can still sign the consent. The 17 year old client is considered a minor, however, since the parents are not available, the emergency exception rule, known as "implied consent" would be followed. The primary healthcare provider must document the nature of the emergency, the reason why immediate treatment is required, and the attempts to obtain consent from the minors parents or legal guardian. 1. Incorrect: The 86 year old client who is disoriented is not considered capable of making an informed decision. 3. Incorrect: Midazolam is a benzodiazepine administered for preoperative sedation/amnesia. For a consent to be legally valid, the consent must be signed prior to being administered preoperative medication or other mind-altering medications. 5. Incorrect: This client with schizophrenia who is hallucinating does not have the ability at this time to understand explanations, understand risks and benefits, and communicate a decision based on that understanding.

What teaching points should the nurse include when educating a client how to prevent a venous stasis ulcer? Select all that apply 1. Elevate legs above heart for 5 minutes, twice a day. 2. Perform leg exercises regularly. 3. Wear graduated compression stockings. 4. Treat itching with prescribed topical corticosteroids. 5. Minimize stationary standing.

2., 3., 4., & 5. Correct: Regular leg exercises improve calf muscle function. Wearing graduated compression stockings will help prevent dilation of lower extremity veins, pain, and a heavy sensation in the legs that typically worsens as the day progresses. Itching can cause the client to scratch, which leads to skin breakdown. Topical corticosteroids can decrease itching and should be used as prescribed. Minimize stationary standing as much as possible to decrease pooling of blood in the lower extremities. 1. Incorrect: Elevate legs above heart for 30 minutes three times a day will minimize edema and reduce intraabdominal pressure.

When explaining to caregivers how to reduce the risk of falls in their elderly parent, the nurse should educate about which measure? Select all that apply 1. Allow the parent to wear shoes that are most comfortable. 2. Assure there is adequate lighting with minimal glare. 3. Use sharply contrasting colors at edges of stairs. 4. Install grab bars beside the shower, tub, and toilet. 5. Encourage the parent to have an inside pet for comfort. 6. Rearrange the furniture for the parent to prevent stagnation.

2., 3., and 4. Correct: Adequate lighting with minimal glare is best to assure there is the amount of illumination needed for safe mobility. Marking the edges of stairs with sharply contrasted colors can help to reduce falls by alerting the elderly client of the change in the elevation of the walkway. The risk of falls in the bathroom can be diminished by installing grab bars to help stabilize the elderly client as they make position changes or transition from the tub, shower, or toilet. 1. Incorrect: Just because the shoes are the most comfortable does mean that they are safe to wear. They may have slick soles, be loose fitting, or have other unsafe issues. Improperly fitting shoes can create a hazard and increase the risk of falls. Unsafe footwear is one of the more common, treatable causes of falls in the elderly. 5. Incorrect: Pets often get in the path of individuals when walking and can create a tripping hazard. Having pets in the home is another one of the more common, treatable causes of falls in the elderly. 6. Incorrect: Elderly individuals manage better when the surroundings are familiar and are kept in the same arrangement. Changes in the environment can increase the risk of falls when objects are no longer where the older adult is accustomed to them being placed.

A client with a total hip arthroplasty (THA) 36 hours ago is scheduled to ambulate in the room. The nurse should initiate which of the following nursing interventions prior to ambulating the client. Select all that apply 1. Keep pressure off heels 2. Assess amount of drainage 3. Instruct on use of mobility aids 4. Encourage flexion hip greater than 90 degrees 5. Teach isometric quadriceps and gluteal setting exercises

2.,3, & 5 Correct: The amount of bloody drainage in the portable suction device should be assessed prior to ambulation. If excess drainage is noted, the ambulation should be postponed and the primary healthcare notified. The safety of the client is a priority. The client should be instructed regarding the proper use of mobility aids before ambulating. The client could fall or not ambulate properly if using the mobility aids in the wrong manner. The nurse should teach the client isometric quadriceps and gluteal setting exercises. The gluteal setting exercise will increase the strength of the gluteal medius which functions to control hip movements. The hamstrings and quadriceps are muscles that are utilized during ambulation. Isometric quadricep exercises will strength the quadriceps which will increase the leg strength. 1. Incorrect: A heal protector or cradle boot can be applied to decrease the pressure on the heel. This is an appropriate intervention for a THA, but the application of the heal protector is not a specific intervention prior to ambulating the client. 4. Incorrect: If the client's hip is flexed more than 90 degrees, there is an increased risk of hip dislocation. The hip should not be flexed greater than 90 degrees before ambulation or after ambulation.

The UAP notifies the nurse that an elderly client seems slightly confused and has become incontinent. Upon assessing the client, the nurse notes an increased pulse with blood pressure lower than normal. What action by the nurse takes priority? 1. Call primary healthcare provider stat. 2. Notify family that client is confused. 3. Have staff collect a urine specimen. 4. Apply oxygen at 2/L via nasal cannula

3. CORRECT: In the elderly, symptoms of urinary tract infections (UTIs) may vary from standard manifestations usually seen in younger client. An older client may initially show neurologic signs such as confusion or falls in addition to frequency, incontinence or lower abdominal pain. Those clients with recurring UTIs may even have a standing prescription written for a urinalysis anytime confusion is noted. 1. INCORRECT: Although the primary healthcare provider will need to be notified about the changes occurring with the client, the nurse should focus on addressing current client needs, including assessing and stabilizing the client if necessary. 2. INCORRECT: The nurse will definitely notify the family when a diagnosis of urinary tract infection is confirmed; however, there is no need to report the confusion to the family until all data has been collected and the primary healthcare provider has determined the outcome. 4. INCORRECT: There is no indication of any respiratory issues requiring the nurse to provide oxygen for the client.

An LPN/VN has been floated to the emergency room following a chemical plant explosion. What task would be best to assign to the LPN/VN? 1. Identify and assess each incoming client. 2. Triage and assign color-coded tags to each client. 3. Gather and apply dressings to open wounds. 4. Initiate oxygen and IV lines as needed.

3. Correct: An LPN/VN's scope of practice includes tasks such as wound care. Covering open wounds will help to decrease bacterial exposure until the registered nurse or primary healthcare provider can assess and treat each wound. If the LPN notes any serious bleeding situations, it would need reported immediately to the RN. 1. Incorrect: Although it will be crucial to identify each incoming client, the LPN/VN's scope of practice does not include assessment. That task would require an RN or primary healthcare provider. 2. Incorrect: In a mass casualty situation, triage allows the nurse or primary healthcare provider to quickly determine which clients are critical versus those stable enough to wait. Because this involves assessment, an LPN/VN would not be assigned this task. 4. Incorrect: Initiating intravenous lines is not within the scope of the LPN/VN. Additionally, the decision to apply oxygen involves assessment of the respiratory system, which also is not within the LPN/VN's scope of practice.

A client diagnosed with an embolic stroke has been admitted to the medical unit. Which nursing assessment would the nurse include to identify an early sign of increased intracranial pressure (ICP)? 1. Bradypnea 2. Bradycardia 3. Restlessness 4. Elevated systolic pressure

3. Correct: An early sign of increased ICP a change in level of consciousness, aggitation, and restlessness. The increased ICP is precipitating neurological changes which results in a decrease in cerebral perfusion. This action results decreased oxygenation to the brain. 1. Incorrect: Bradypnea is a late sign of increased ICP. The normal breathing rate for an adult is 12 - 20 breaths per minute. Increased intracranial pressure will compromise the blood flow in the brain. The result of the decreased blood flow is bradypnea. 2. Incorrect: As the intracranial pressure increases and hypertension occurs the parasympathetic system is stimulated. The parasympathetic system stimulation results in bradycardia. Bradycardia is a pulse less than 60 beats per minute. Bradycardia is not an early sign of increased ICP. 4. Incorrect: The cerebral blood flow decreases as intracranial pressure elevates. The response is an increase in the systolic pressure as result of the arterial pressure increase. This is a later sign of increased ICP.

A client who has had a stroke presents with lethargy, facial droop, and slurred speech. The client has a history of gastroesophageal reflux disease (GERD). From this history, what does the nurse recognize as a priority risk for this client? 1. Diminished colonic motility 2. Esophageal hemorrhage 3. Aspiration pneumonia 4. Stress ulcers

3. Correct: Anyone who has had a stroke is at risk for aspiration, especially with a history of reflux disease. It is important to remember that the stomach is full of acid, so when aspiration of this acid occurs, it causes irritation to the lung tissue. The client can develop severe pneumonitis. That's what could kill the client, so this answer takes priority. 1. Incorrect: Colonic motility is a disorder with the motility of the colon that presents with diarrhea or constipation. Diminished colonic motility may become a problem for this client, but aspiration pneumonia is more acute. Remember airway, breathing, and circulation will take priority. 2. Incorrect: Esophageal hemorrhage is seen with esophageal varices, not reflux disease. This option does not apply to this question. 4. Incorrect: GERD is not associated with increased risk for stress ulcers, but GERD can also lead to strictures and/or precancerous lesions called Barrett's esophagus. Though stress ulcers could occur in this client, they are not priority over the airway and are not related to this client's history!

The nurse is caring for a client admitted with heart failure associated with an acute MI. At which time point did the nurse begin to intervene incorrectly? Exhibit 1. 1115 2. 1120 3. 1125 4. 1130

3. Correct: At 1125, the nurse failed to follow protocol for nitroglycerin infusion. The nurse increased the IV rate by 6 mL/hr (going from 10-20 mcg/min). 1. Incorrect: The nurse mixed the nitroglycerin appropriately and connected the tubing at the correct IV site. The infusion rate was started at 3 mL/hr which delivered the appropriate starting dose at 5 mcg/min. 2. Incorrect: At 1120 the client is still hurting and the BP is above 120 systolic, so the nitroglycerin infusion can be increased by 5 mcg/min which would increase the rate to 6 mL/hr. 4. Incorrect: At 1130 the client is still hurting and the BP is above 120 systolic, so the nitroglycerin infusion can be increased by 5 mcg/min which would increase the rate to 15 mL/hr.

A client at 34 weeks gestation with pregnancy induced hypertension (PIH) reports "heartburn." Which action by the nurse has priority? 1. Administer an antacid per standing orders. 2. Check client's blood pressure. 3. Call the primary healthcare provider immediately. 4. Assure client this is a normal discomfort of pregnancy.

3. Correct: Epigastric discomfort is commonly described as "heartburn" by pregnant clients, but epigastric discomfort is a symptom of impending rupture of the liver capsule and seizures associated with worsening PIH and eclampsia. As a new nurse we need to assume the worst. Call the primary healthcare provider. 1. Incorrect: Not a concern as much as impending seizure symptoms. Administering an antacid will not fix the problem if PIH is worsing. This is delaying care. 2. Incorrect: Not a concern as much as impending seizure symptoms. Checking the client's blood pressure is not the priority in this situation. It will not fix the problem. 4. Incorrect: Not in this situation. Heartburn is a normal discomfort or right upper quadrant pain in a client with PIH may indicate impending rupture of the liver capsule which is a life threatening complication.

Which statement by a client diagnosed with infectious mononucleosis indicates to the nurse that education has been successful? 1. "I should let my primary healthcare provider know if I start having pain in the side of my stomach" 2. "I can return to my normal activities in 5 days." 3. "I will not let others drink from my glass." 4. "My immediate family needs to get vaccinated against mononucleosis."

3. Correct: Infectious mononucleosis, caused by the Epstein-Barr virus is transmitted by saliva and intimate physical contact like kissing, sharing of utensils, and eating/drinking after others. 1. Incorrect: The client should observe for left upper quadrant abdominal pain radiating to the left scapula as this is an indicator of splenic rupture, a complication of infectious mononucleosis. 2. Incorrect: This is too soon. Most people get better in 2 to 4 weeks; however, some people may feel fatigued for several more weeks. Occasionally, the symptoms of infectious mononucleosis can last for 6 months or longer. 4. Incorrect: There is no vaccine to protect against infectious mononucleosis. The best way of the Epstein-Barr virus is to eliminate contact with oral secretions.

A new nurse asks the charge nurse for assistance in interpreting arterial blood gases (ABGs) for a client. What acid/base imbalance should the charge nurse tell the new nurse these ABGs indicate in the client? 1. Partially compensated metabolic acidosis 2. Partially compensated respiratory alkalosis 3. Partially compensated metabolic alkalosis 4. Partially compensated respiratory acidosis

3. Correct: Partially compensated metabolic alkalosis is indicated by these ABGs. The pH is 7.5 (normal 7.35-7.45) which is high, which means alkalosis. The PaCO2 is 58 (normal 35-45) which is high. Greater than 45 is acidosis from too much CO2. The HCO3 is 35 (normal 22-26) which is high. A high bicarb level equals alkalosis. The HCO3 matches the pH as both indicate alkalosis. The initial problem was a kidney problem or metabolic alkalosis. The lungs are trying to compensate by holding on to more acid. So the correct answer is Option 3: Partially compensated metabolic alkalosis. 1. Incorrect: A pH of greater than 7.45 indicates alkalosis rather than acidosis. So this option is incorrect. 2. Incorrect: The PaCO2 would be low rather than high if the problem was respiratory alkalosis. 4. Incorrect: A pH of greater than 7.45 indicates alkalosis rather than acidosis. So this option is incorrect.

A client who had a triple lumen catheter placed in the right subclavian vein 30 minutes ago reports chest discomfort and shortness of breath. The assessment reveals BP 92/58, HR 104, Resp 28, and unequal breath sounds over lung fields. What problem should the nurse suspect this client is exhibiting? 1. Myocardial infarction 2. Atelectasis 3. Pneumothorax 4. Pneumonia

3. Correct: Pneumothorax is the number one potential complication of all central venous access devices. Signs and symptoms include chest pain, dyspnea, shoulder or neck pain, irritability, palpitations, light-headedness, hypotension, cyanosis, and unequal breath sounds. 1. Incorrect: The hints point to a pneumothorax rather than an MI. The triple lumen catheter and unequal breath sounds are the biggest hints. 2. Incorrect: Atelectasis is the collapse of alveoli and is caused by a blockage of the air passages or by pressure on the outside of the lung. Examples of causes of atelectasis are mucus that plugs the airways, anesthesia, pleural effusion, prolonged bedrest with few position changes, and shallow breathing. 4. Incorrect: Pneumonia is an infection that causes inflammation of the air sacs in one or both lungs. The air sacs may fill with fluid or pus (purulent material), causing a cough with phlegm or pus, fever, chills, and difficulty breathing.

The nurse is helping a UAP transfer a bed-fast client from a litter into the bed. What is the nurse's priority action? 1. Verify the client's identity band is correct. 2. Pull curtain to protect the client's privacy. 3. Lock wheels on both the litter and the bed. 4. Use a transfer board to move client safely.

3. Correct: Safety is a priority with clients at all times and in all situations. When transferring a client, the nurse must be certain there is no potential for harm to either staff or the client. In this scenario, two individuals are moving the client from flat surface to flat surface. Therefore, the priority is to be certain neither surface can move during the transfer by making certain all wheels are locked on both litter and bed. 1. Incorrect: Although it is important to be certain staff has the correct client for that room, another issue takes priority in this situation. 2. Incorrect: Client privacy is always of concern and should be observed during any nursing activity, particularly when transporting or moving the individual. However, a greater concern exists. 4. Incorrect: A transfer board is a device similar to a small surfboard which is placed under the client's back, allowing staff to carefully and safely slide a client between two surfaces. However, such a device is not always mandatory, nor even available. A client can be moved without such equipment.

The nurse is caring for a client following a transurethral resection of the prostate (TURP). The client has a 3 way irrigation catheter in place. Which observation would indicate the need to slow the irrigation? 1. Clots in urine 2. Bladder pressure 3. Clear urine 4. Bladder spasms

3. Correct: The irrigation is regulated so that the urine is free of clots and slightly pink tinged. When it becomes clear after surgery, the fluid is going too fast and not clearing any blood clots effectively. 1. Incorrect: The irrigation should be increased if you see clots in order to keep the catheter patent. 2. Incorrect: Bladder pressure may mean that the indwelling urinary catheter is obstructed. Either increase flow or manually irrigate the catheter to ensure patency and no retention of fluid in the bladder. 4. Incorrect: Bladder spasms occur with clots, so you do not want to slow the irrigation if this happens. This would indicate the need for increasing the irrigation fluid rate.

A client has just found out that she is pregnant and asks the nurse, "When is my baby due?" The client's last menstrual period began March 3. What date will the nurse calculate as the expected date of confinement? 1. December 3 2. December 7 3. December 10 4. December 13

3. Correct: The most common method of determining the expected date of confinement is by Nagele's rule. To use this method begin with the first day of the last menstrual period, add seven days, subtract 3 months and add one year. So the expected date of confinement for this client would be December 10. 1. Incorrect: To use Nagele's rule begin with the first day of the last menstrual period, add seven days, subtract 3 months and add one year. So the expected date of confinement for this client would be December 10. 2. Incorrect: To use Nagele's rule begin with the first day of the last menstrual period, add seven days, subtract 3 months and add one year. So the expected date of confinement for this client would be December 10. 4. Incorrect: To use Nagele's rule begin with the first day of the last menstrual period, add seven days, subtract 3 months and add one year. So the expected date of confinement for this client would be December 10.

What action should the nurse take when testing a client's near vision? You answered this question Incorrectly 1. Have client read a Snellen chart from 20 feet away. 2. Have client read Ishihara plates at 30 inches (75 cm). 3. Have client read a newspaper at 14 inches (36 cm). 4. Have client alternate gaze from a near object to a distant object.

3. Correct: The nurse can get a general idea of near visual acuity by asking the client to read from a newspaper. The newspaper should be held 14 inches from the eyes. This exam can also be done with the Jaeger chart containing a few short lines or paragraphs of printed text. The size of the print gradually gets smaller. The client is asked to hold the card about 14 in. (36 cm) from the face and read aloud the paragraph containing the smallest print he/she can comfortably read. Both eyes are tested together, with and without corrective lenses. This test is routinely done after age 40, because near vision tends to decline as one ages (presbyopia). 1. Incorrect: The Snellen chart is used to test distant vision. To test distance vision, individuals stand 20 feet from the Snellen eye chart, cover one eye, read aloud the smallest line they can clearly see, and then repeat this process with the other eye. After performing an eye test, a person's visual acuity is written as a fraction. Normal vision is defined as 20/20 visual acuity, which means at 20 feet away from the eye chart, the person is able to read the line that most human beings with normal vision can read at 20 feet away.​ 2. Incorrect: Ishihara plates are used to assess color blindness, and are not used to test near vision. 4. Incorrect: Having the client alternate their gaze from a near object to a distant object tests for accommodation. It does not test for near vision.

The nurse manager on a medical-surgical unit receives official notification that staff overtime must be decreased as a cost-saving measure. In order to reorganize staffing, the nurse manager should initiate which action first? 1. Announce the new changes at the monthly staff meeting. 2. Ask for any staff objections to rearranging work hours. 3. Invite staff to contribute ideas on scheduling changes. 4. Explain administration is demanding a decreased overtime.

3. Correct: The nurse manager is aware that open communication with staff is vital to increase workplace satisfaction and staff retention. One important aspect is encouraging the flow of ideas between management and staff members. Open communication and brainstorming sessions in which staff can freely share thoughts or ideas creates a positive work environment while helping decrease dissatisfaction. 1. Incorrect: While it is true that the nurse manager is ultimately responsible for implementing and announcing new schedule changes, doing so without any staff input can create discontent in the work environment. When staff do not feel vested in any new process, there is a sense of underappreciation. This perceived lack of control can create distrust and frustration among personnel, ultimately impacting client care. 2. Incorrect: Although this action appears to be opening lines of communication, the nurse manager is actually fostering animosity in a situation where the outcome is already pre-determined. Allowing staff to vent is acceptable but the nurse manager should focus on constructive methods of adjustment to the impending mandated changes. 4. Incorrect: The nurse manager is aware that health care facilities often face both political and financial issues that impact staff and clients simultaneously. The responsibility of the nurse manager is to implement change in a positive manner, while assisting staff adaptation even to unpopular modifications. Assigning blame for the changes to administration will not help staff adjust.

A client has been taking tranylcypromine for approximately two weeks. The client is visiting the nurse at the local mental health center for follow up and group therapy. Which client comment indicates a lack of understanding of the medication that could result in a medical emergency? 1. I know that I must take this medication until my primary healthcare provider tells me to stop. 2. It is frustrating to have to follow dietary restrictions. 3. I am getting a cold, and I am going to take some over the counter cold medicine. 4. I am going to have broccoli salad and roasted turkey for lunch today.

3. Correct: This is an MAOI medication. OTC cold medications could result in hypertensive crisis when combined with the monoamine oxidase inhibitor. Warnings are placed on cold preparations and other medicines that are not to be taken with the MAOIs. Be cautious about adding over the counter medications and possible drug interactions. 1. Incorrect: This is a true statement. The primary healthcare provider's orders should be followed. This comment would not result in a medical emergency. 2. Incorrect: Foods high in tyramine should be avoided while taking tranyclpromine, and dietary restrictions must be followed. This comment indicates correct understanding of this medication. 4. Incorrect: This dietary choice demonstrates adequate understanding of the medication and dietary restrictions required.

What would be the nurse's priority for a child who has arrived at the emergency department after sustaining a severe burn? 1. Start intravenous fluids. 2. Provide pain relief. 3. Establish airway. 4. Place an indwelling catheter.

3. Correct: This stem does not tell you where the child's burns are. However, you are told that the burns are severe. So assume the worse. What are you most worried about the child losing? Yes, the airway. So we want to assess, establish, and maintain an airway. 1. Incorrect: Not before airway. This child will need IV fluid resuscitation within the first 24 hours. But if you can only pick one action to complete first, it better be to make sure the airway is patent. Then you can start the IV or delegate the task to someone else. 2. Incorrect: Give pain med after starting the IV, not before airway. The best pain relief method for a severe burn is going to be through the IV route. But we must make sure the airway is patent first. 4. Incorrect: Intake and output will need to be closely monitored in the client who is severely burned. But again, it will need to be done after establishing a patent airway.

What should the nurse emphasize when teaching clients how to decrease the risk of chronic obstructive pulmonary disease? Select all that apply 1. Avoid exposure to individuals with respiratory infections. 2. Increase intake of Vitamin C. 3. Eliminate exposure to second hand smoke. 4. Avoid prolonged exposure to occupational dusts and chemicals. 5. Get a yearly influenza and pneumococcal vaccination.

3., & 4. Correct: The most important environmental risk factor for COPD is cigarette smoking. Second hand smoking also contributes to COPD. Risk factors for COPD include prolonged and intense exposure to occupational dust and chemicals as well as indoor and outdoor air pollution. 1. Incorrect: Exposure to individuals with respiratory infections does not increase risk of chronic obstructive pulmonary disease. Respiratory infections may cause an acute exacerbation in a client with existing COPD. 2. Incorrect: Increasing intake of vitamin C does not decrease risk of obstructive pulmonary disease. 5. Incorrect: Clients should get the influenza vaccine annually in autumn. The pneumococcal vaccine should be administered every 5 years, rather than yearly.

The nurse asks the UAP why she is crying in the hallway. The UAP states, "That client in room 107 is so mean. No matter what I say they just yell at me." Which is the best response by the nurse? 1. "That client is like that to everyone. He will be discharged soon." 2. "Just don't go back in the room the rest of your shift." 3. "I know you are upset. You are trying to provide care to them." 4. "Let's go tell the charge nurse to fix this problem."

3.CORRECT. Acknowledging the UAP's feelings and offering positive feedback is the best response. It is a therapeutic response. 1.INCORRECT. This option is pushing away the client's behavior and it does not acknowledge the upset UAP. This is not a therapeutic response. 2.INCORRECT. Telling the UAP to not care for the client anymore is not correct. 4.INCORRECT. Going to tell the charge nurse is like delaying care of the situation. This option does not allow you to do anything for the UAP that is upset.

The emergency room nurse is assessing a client reporting severe abdominal pain for several hours prior to arrival at the hospital. Assessment findings include slight mottling of the lower extremities and a pulsating mass near the umbilicus. Which actions should the nurse implement immediately? Select all that apply 1. Position client on the left side. 2. Apply warm blankets to legs. 3. Administer I.M. pain medication. 4. Alert the operating room staff. 5. Notify the primary healthcare provider. 6. Palpate mass to determine size.

4. & 5. Correct: The client's symptoms indicate the presence of an aortic abdominal aneurysm that may be dissecting (rupturing) at this time. This is a life-threatening emergency and the client will need urgent surgery to survive. The nurse should immediately notify the healthcare provider and alert the operating room staff of impending surgery. 1. Incorrect: These are the classic symptoms of a dissecting abdominal aneurysm, a life-threatening situation requiring immediate surgery. Positioning the client on either side is contraindicated as that action may cause further internal bleeding, complete rupture of the aneurysm, or death. 2. Incorrect: Mottling of lower extremities accompanied by severe abdominal pain suggests a dissecting abdominal aneurysm. The discoloration of lower extremities indicates compromised circulation secondary to interrupted blood flow because of the aneurysm. This client would not benefit from warm blankets but rather needs immediate surgery to survive. 3. Incorrect: Pain medications in general are not administered until an exact diagnosis is confirmed, since relieving pain would mask those signs or symptoms needed to verify the problem. While the client may be given medications at some point, this is not the life-saving action the nurse must take immediately. 6. Incorrect: The client's symptoms are suggestive of a dissecting abdominal aneurysm, a life-threatening emergency requiring immediate surgical intervention. It is never acceptable for the nurse to palpate an abdominal mass, particularly a pulsating mass, since this would likely cause complete rupture of the blood vessel and immediate death.

A client awaiting discharge for a broken left tibia is to be sent to physical therapy for crutches and crutch walking. The client reports having brought a pair of crutches borrowed from a family member. What is the most appropriate action for the nurse to take now? 1. Cancel physical therapy and allow client to leave. 2. Ask client to stand with crutches to check the size. 3. Tell client insurance will not permit use of old crutches. 4. Send client with crutches to physical therapy for evaluation.

4. CORRECT. The physical therapy department is best qualified to assist a client in adjusting to the use of crutches prior to discharge. Because the client wants to use older crutches, it is even more important for a physical therapist to determine whether it is safe for the client to do so. Physical therapy can evaluate the condition of the old crutches, the client's ability to manage that equipment and to walk safely with those crutches. 1. INCORRECT. It is permissible for a client to use previously owned medical equipment. However, the stability of that equipment and the client's ability to use the equipment safely must be evaluated by physical therapy. Cancelling physical therapy would also violate the physician's orders and place the client at risk for injury upon discharge. 2. INCORRECT. While the nurse may be able to adjust the old crutches to the client's height, crutch safety and walking should be evaluated by physical therapy to be certain the previous equipment is appropriate. 3. INCORRECT. Insurance does not designate whether assistive medical devices can be reused by clients or if a new device must be purchased. It is cost effective to reuse durable medical equipment if it is appropriately suited to the client's current needs.

A nurse has received morning report on multiple clients. What client should the nurse assess first? 1. Client on 2/L min, of oxygen by nasal cannula with pneumonia. 2. Client with Crohn's disease reporting two semi-loose stools. 3. Client one day post-appendectomy reporting abdominal cramps. 4. Client on heparin drip reporting bleeding gums when brushing teeth.

4. CORRECT: This client is currently on a heparin drip, possibly for a DVT. Bleeding gums sometimes occurs in those who brush teeth too vigorously; however, bleeding in a client on a heparin drip could indicate a serious side effect. This client should be seen immediately. 1. INCORRECT: It is not unusual for a client with pneumonia to need supplemental oxygen by nasal cannula and 2L/min, is a very small amount. Although this client will definitely need a respiratory assessment, there is no indication that any complications are occurring. This client is not a priority. 2. INCORRECT: Crohn's disease is an auto-immune process in which inflammation of the large and small bowel causes pain and diarrhea. Reporting loose stools would not be unexpected for this client; therefore, this client would not need to be the first assessment. 3. INCORRECT: One-day after an appendectomy, it would not be unusual for a client to have some abdominal pain or cramping. Although the nurse will need to assess for bowels sounds and inspect the dressing, this client is not a priority.

Which statement by a client would indicate to the nurse that education about alendronate has been successful? 1. "It is recommended that I recline for 15 minutes after taking my medication." 2. "Food should be eaten immediately after taking alendronate." 3. "My medication tablet should be chewed for rapid absorption." 4. "I should drink a full 8 ounce glass of water with my medication."

4. Correct: Alendronate is a biophosphonate drug used in the treatment of osteoporosis and other bone diseases. The client should take each tablet in the morning with a full glass of water (6-8 ounces or 180-240 ml) at least 30 to 60 minutes before the first food, beverage or medication of the day, to increase absorption. 1. Incorrect: After taking alendronate, the client should remain upright (sitting or standing) for 30-60 minutes. The client should not lie down until after eating. These actions help to decrease the likelihood of esophageal and GI associated side effects. 2. Incorrect: The client should wait at least 30-60 minutes before eating, drinking or taking any other medication, to increase absorption. 3. Incorrect: The client should not chew the medication tablet, mouth ulcers can occur when alendronate is chewed or dissolved in the mouth.

A tour bus is involved in an accident, sending several clients to the emergency room (ER) for treatment. An unconscious client with multiple internal injuries requires immediate surgery. When itemizing the client's belongings, the nurse finds a wallet containing four thousand dollars. What is the appropriate method for the nurse to secure the money? 1. Place wallet inside client's pants and then in belongings bag. 2. Secure the money in an envelope in the ER narcotics drawer. 3. Sign money over to the hospital CEO until client is discharged. 4. Tally cash with 2nd nurse, document and lock in hospital safe.

4. Correct: All personal valuables in the possession of an unconscious client, including money or jewelry, must be tallied in the presence of two nurses and then documented in the client's main chart. Valuable items such as watches, rings or necklaces must also be secured until a family member is contacted, or the client is able to designate disposition of same. With large amounts of cash, a passport or other such important items, it is vital to account for and secure those items until returned to client or family. When dealing with money, two nurses must count the cash and document the total on the client's chart. The funds are then locked in the main hospital safe until the client is discharged or delegates a family member to retrieve same. 1. Incorrect: Even though the client's belongings bag is personal property, it is not a secure location. The bag is usually kept in the client's room or closet which does not provide security for a large amount of money. 2. Incorrect: While locking the cash into the ER narcotics drawer may be a temporary solution during care of the client, this is not an adequate long-term solution. The client will be sent to the operating room, and then admitted to a room. The money is personal property which should remain with the client in a secured manner. 3. Incorrect: Entrusting the funds to a single individual, even the facility CEO, is not the appropriate method of securing valuables.

The hospice nurse has been assigned a new client who is being cared for at home by family members. Based upon the client's physical assessment, the nurse is aware that the client's death is imminent. What is the nurse's most important role in the care of the family at this time? 1. Providing care for the client, allowing the family to rest. 2. Providing education regarding the symptoms the client will likely experience. 3. Allowing the family to express their feelings and actively listening. 4. Communicating the client's impending death to the family while they are together.

4. Correct: Communicating news of the client's impending death to the family while they are together. The nurse's most important role in the care of the family is compassionate communication. The family needs to be informed about the situation so that they are prepared for the client's death and can provide support to one another. 1. Incorrect: Providing respite time when death is imminent is not a priority. Family should be allowed to spend time with the client. They will, more than likely, want to be with the client in the last hours. 2. Incorrect: When death is imminent, education of what to expect is appropriate, but does not take priority over compassionate communication. Compassionate communication is most important at this time. 3. Incorrect: Silence and listening sends a message of acceptance and comfort. Although important, allowing for expression of feelings is not more important than preparing the client for the imminent death.

A newly married wife tells the nurse, "I told my husband that I may not know how to cook, but I can sure do the dishes!" Which defense mechanism is the client displaying? 1. Projection 2. Displacement 3. Sublimation 4. Compensation

4. Correct: Compensation is consciously or unconsciously overemphasizing a characteristic to compensate for a real or imagined deficiency. Making up for deficits in one area by excelling in another helps to raise or maintain the client's self-esteem. 1. Incorrect: Projection is attributing one's own thoughts or impulses to another person as if they had originated in the other person. This is unconsiously done and usually includes intolerable wishes and emotional feelings. 2. Incorrect: Displacement is shifting or transferring the emotional element of a situation from a threatening object to a non-threatening object. This could include transferring emotions from a person, object or situation to another person, object or situation. 3. Incorrect: Sublimation is redirecting a socially unacceptable impulse into socially acceptable behavior. Examples include strong aggressive or sexual drives.

How does the nurse identify the correct size of crutches for a client? 1. Turn the crutches upside down and measure from the heel to the shoulder. 2. Obtain a set of crutches and adjust the height until the client can stand comfortably while resting the axilla on the crutch pad. 3. Measure the client while standing upright from the axilla to the heel then adjust the crutches so that the elbow flexion is a 30-degree angle. 4. Measure the client from 2 inches below the axilla to 6 inches lateral to the client's heel.

4. Correct: Measuring the client from 2 inches below the axilla to 6 inches lateral to the client's heel correctly measures a client for crutches. This is the correct size while a client is standing. 1. Incorrect: This is not the correct way to choose the correct size crutches. Without the proper fit safety is a concern. 2. Incorrect: This is not how to choose the correct size of crutches. The client should not rest their weight on the crutch pad as this can cause damage to the brachial plexus nerve. 3. Incorrect: This is not how to choose the correct size of crutches. The shoulders should be relaxed, the hand piece should be adjusted to provide a 20°- 30° elbow flexion. The 2 inch drop below the axilla allows the weight to be pressed against the sides and the hands absorb the weight. The crutch should not be placed against the axilla or the brachial plexus nerve could be damaged.

A post-operative client has received morphine for pain. The nurse re-assesses the client 10 minutes later. Which assessment data warrants further action by the nurse? B/P 110/76, Pulse 68, Respirations 8, Pain level of 5, dressing dry and intact. 1. Blood pressure 94/60 2. Pulse rate 72/min 3. Pain level 3/10 4. Respiratory rate at 8/min

4. Correct: Normal respiratory rate is 12-20 per minute. The respiratory rate indicates respiratory depression following administration of an opioid. Care should be taken to titrate the dose so that the patients pain is controlled without depressing the respiratory function. 1. Incorrect: Respiratory rate warrants immediate action. However, blood pressure will continue to be monitored. 2. Incorrect: Pulse rate warrants no further action. Pulse rate is normal. 3. Incorrect: The pain level is expected following surgery. The client should continue to have a reduction on pain, as 10 minutes is not long enough to fully evaluate.

What instruction would the nurse give a client about a newly prescribed salmeterol inhaler? 1. "Use the inhaler immediately if wheezing and shortness of breath occur during exercise." 2. "Use the inhaler when you experience a stuffy nose due to seasonal allergies." 3. "Carry the inhaler with you at all times and take 2 puffs anytime you experience an exacerbation." 4. "This inhaler should be used routinely as prescribed even when free of symptoms."

4. Correct: Salmeterol is a maintenance medication. It can prevent asthma attacks and exercise induced bronchospasm. Salmeterol acts as a bronchodilator. It works by relaxing muscles in the airways to improve breathing. 1. Incorrect: Salmeterol can be used to help prevent exercise induced bronchospasm, but it should be taken 30 to 60 minutes before exercise. 2. Incorrect: Salmeterol is indicated for asthma only and bronchospasm induced by obstructive pulmonary disease. It is not indicated for seasonal allergies. 3. Incorrect: Salmeterol is a maintenance medication. Albuterol is used as a "rescue inhaler" for bronchospasms.

During evening rounds on a medical unit, a client is discovered in cardiac arrest. After activating the code button, the nurse initiates chest compressions. A second nurse enters the room to assist. What priority task could be delegated to the second nurse? 1. Retrieve the crash cart. 2. Document the code events. 3. Notify the primary healthcare provider 4. Begin oxygenating the client.

4. Correct: The chest compressions, airway, and breathing (CAB) sequence is always of primary concern. The first nurse correctly activated a code and then began chest compressions. The second nurse will assist by oxygenating the client, using a bag valve mask. 1. Incorrect: Although it will be necessary to bring the crash cart into the room, the initial priority should focus on the client's needs. In the case of a client in cardiac arrest, the first personnel to respond must focus on CPR protocols, including compressions and oxygenation. Other personnel can bring the crash cart into the room. 2. Incorrect: Documenting all the events that occur during a code is vital for both legal and quality assurance purposes; however, the initial priority must focus on stabilizing the client. 3. Incorrect: The Healthcare Provider does need to be notified, but it is not an immediate priority for either nurse. Ancillary personnel, such as the unit secretary, can complete this task. Professional personnel must focus on the client's immediate needs at this critical point.

An elderly homeless client is brought to the emergency room for evaluation following a fall. What assessment findings by the nurse should be reported immediately to the primary healthcare provider for further evaluation? 1. The client is unsteady when walking to the bathroom. 2. The client cannot state day, date or present location. 3. The client refuses to remove either shoes or socks. 4. The client has loss of sensation below the left knee.

4. Correct: The elderly client has several issues which are concerning to the nurse. However, with no specific health history, lack of sensation to an extremity can signal severe circulatory damage or an undiagnosed problem such as diabetes. The primary healthcare provider should immediately be made aware of the need to evaluate this further. 1. Incorrect: An unsteady gate is cause for concern, particularly in an elderly client, because of the potential for a fall. However, there is another situation of even greater concern at this time. 2. Incorrect: It is not unusual for an elderly individual to lose track of day, date or time, particularly in a situation such as homelessness. The nurse may want to investigate why the client is not even aware of the hospital setting, but this is not the first concern. A more serious issue is present. 3. Incorrect: Refusal to remove any type of clothing is a client right, and does not require an immediate call to the primary healthcare provider unless there is a suspected problem in that area of the body covered by the clothing. This situation can be addressed but following the priority concern.

Which client in the emergency department should the nurse identify as being the highest priority? 1. Client with emphysema reporting shortness of breath. 2. Client with a cut on the left calf with moderate bleeding. 3. Client with onset of confusion 1 hour prior to arrival. 4. Client with facial swelling and rash after taking azithromycin.

4. Correct: Were you able to recognize this as a probable reaction to the antibiotic that was taken? Next, it is important to note that there is not only a rash, but facial swelling is also present. That should alert the nurse to the possibility that there could be a rapid onset of airway swelling that could cause airway occlusion. This makes this client the priority over the other clients. 1. Incorrect: It is not uncommon for clients with emphysema to experience shortness of breath. This is a concern, and the client needs additional measures to help relieve the shortness of breath, but is not as likely to result in a rapid airway occlusion as the allergic reaction could. 2. Incorrect: Again, this is a situation that will require intervention. It pertains to circulation because there is moderate bleeding from a cut. However, this does not take priority over the airway. 3. Incorrect: We are concerned about this client and neuro checks will need to be performed. This could be many various things including a stroke, effects of medications or drugs, or other neurological conditions. But, this does not take priority over the airway.

A client with a history of syncope and transient arrhythmias has been ordered a Holter monitor for 48 hours. The nurse knows that teaching has been effective when the client makes what statement? 1. No follow up care will be needed after the monitor is removed. 2. It is okay to shower or bath while wearing this equipment. 3. I have to take it easy and not exercise for the next two days. 4. It's important to write down all my activities during this time

4. Correct:The purpose of the Holter monitor is to detect cardiac irregularities over an extended period of time, in this case 48 hours. Although the monitor will record heart rate and rhythm for two days, it is vital for the client to keep a log or diary during that time, indicating the precise time and type of every activity. Additionally, this log needs to indicate any chest pain or palpitations the client experiences during that time, to assist the primary healthcare provider in diagnosing cardiac dysfunctions. 1. Incorrect: A Holter monitor is a mobile diagnostic test utilized by the cardiologist to help determine a cause for this client's syncopal episodes or arrhythmias. Once the client has the monitor and electrodes removed, the primary healthcare provider will analyze the data before meeting with the client to discuss the findings. Regardless of any suggested treatment options, the cardiologist needs a follow up visit with the client. 2. Incorrect: Showering or tub bathing is not permitted while wearing the Holter monitor as this may interfere with the functioning of the equipment. Only a careful sponge bath is permitted. Clients are also instructed to avoid heavy machinery, electric razors, microwave ovens and even hair dryers since can also affect accuracy and performance of the monitor. 3. Incorrect: The purpose of wearing Holter monitor for 24-48 hours is to diagnose cardiac arrhythmias during ADL's or exercise. The client cannot remove the monitor at any time during that period since that would cause inaccurate readings, or even the loss of valuable data. The client is instructed to complete all routine daily activities during that time, including work or exercise, to help identify actions that contribute to the symptoms or cardiac irregularities.

A client with a history of congestive heart failure (CHF) has been admitted with digoxin toxicity. After reviewing the initial laboratory results, the nurse knows what abnormal findings most likely contributed to the digoxin toxicity? Exhibit Select all that apply 1. Sodium 2. Calcium 3. Albumin 4. Potassium 5. Magnesium

4., & 5. Correct: Hypokalemia and hypomagnesemia both can increase the client's potential to develop digoxin toxicity. Digoxin and potassium both bind at the same location on the ATPase pump. When potassium levels are low, more digoxin will attach to the sites, leading to toxicity. Low magnesium levels sensitize the cardiovascular system to the toxic effects of digoxin. 1. Incorrect: The presence of digoxin in the body does slightly inhibit the activity of the NA/K+ pump. However, even though the sodium level is slightly elevated, there is no direct correlation between that increased sodium level and digoxin toxicity. 2. Incorrect: A calcium level of 9.9 is within the normal limits of 9.0 to 10.5 mg/dL (2.25-2.62 mmol/L). Calcium is controlled by the parathyroid glands, generally shifting between the bones and serum. A normal calcium level would not contribute to digoxin toxicity. 3. Incorrect: Albumin is a protein synthesized by the liver which helps to maintain fluid within the vascular spaces and transport soluble products throughout the body. This specific laboratory result is within normal limits. Nonetheless, albumin has no effect on digoxin levels in the body.

What is the best way for the nurse to maintain client privacy? 1. Allow the family members to remain in the room during assessment. 2. Answer client questions from family members just outside the client's room. 3. Remain logged into the bedside computer for quick access in case of an emergency. 4. Discuss client information with the healthcare provider in the nurses' station.

4.CORRECT. Discussing client specific information is best shared in the nurses' station so other clients and visitors will not overhear the discussion. 1.INCORRECT. The client should be asked if it is okay for the visitors to remain in the room during assessment. The client may have problems or concerns that they do not want a visitor to know. 2.INCORRECT. Family questions should be addressed inside the client's room or in another private location. If questions are answered just outside of the room, then anyone can overhear the conversation in the hallway. 3. INCORRECT. The staff should always sign completely out of the computer when leaving the room. If not, anyone can have access to any client records


Kaugnay na mga set ng pag-aaral

politics of environmental issues fall 2021

View Set

Sports Leadership Exam 2 Study Guide

View Set

NURS 302 - ATIs and study guide questions

View Set

US History Unit 5 Flash Cards (Combine)

View Set

NURS 302 Module 4: Bipolar, Schizophrenia & Psychotic disorder - practice questions

View Set

Leadership Hesi Adaptive Quizing

View Set